Сохранен 527
https://2ch.hk/sci/res/275146.html
24 декабря Архивач восстановлен после серьёзной аварии. К сожалению, значительная часть сохранённых изображений и видео была потеряна. Подробности случившегося. Мы призываем всех неравнодушных помочь нам с восстановлением утраченного контента!

Математика, тред №20

 Аноним 14/06/15 Вск 00:49:02 #1 №275146 
14342321427540.jpg
Восстановим старую добрую традицию и начнём юбилейный тред с задачи:

Найти размерность векторного пространства действительных чисел над полем рациональных чисел. Ну и доказать, что векторные пространства действительных и комплексных чисел над полем рациональных чисел - изоморфны.
Аноним 14/06/15 Вск 00:51:18 #2 №275148 
Предыдущий: >>271609
Аноним 14/06/15 Вск 01:15:04 #3 №275150 
>>275146
>Найти размерность векторного пространства действительных чисел над полем рациональных чисел. Схерали действительные числа - это векторное пространство?
Аноним 14/06/15 Вск 01:29:31 #4 №275152 
>>275146
Ну не больше чем счетная и не конечная.
=> счетная
Аноним 14/06/15 Вск 01:33:14 #5 №275153 
>>275150
Потому что удовлетворяет аксиомам векторного пространства.
Аноним 14/06/15 Вск 02:12:56 #6 №275154 
>Найти размерность векторного пространства действительных чисел над полем рациональных чисел.
Не существует базиса => нет и размерности.
>Ну и доказать, что векторные пространства действительных и комплексных чисел над полем рациональных чисел - изоморфны.
Не изоморфны.
мимо-не-люблю-аксиому-выбора
Аноним 14/06/15 Вск 02:43:41 #7 №275158 
>>275154
Кстати, если уж я вспомнил про это. Собственно это довольно занятная задачка, несколько сложнее оригинальной ОП-овской, но все-равно вполне посильная. Пусть аксиома выбора не верна и более того, все подмножества вещественной прямой измеримы по Лебегу. Доказать, что
1. У R, как векторного пространства над Q, нет базиса;
2. R и С не изоморфны, как векторные пространства над Q.
Аноним 14/06/15 Вск 02:54:21 #8 №275159 
>>275154
>не-люблю-аксиому-выбора
Почему?
Аноним 14/06/15 Вск 03:08:01 #9 №275161 
В чем секрет успехов еврейских математиков? Даже если ученый и не признает себя евреем, то в нем все-равно что-то заложено из этой культуры, особое мышление может. Алсо в вводной лекции по теории игр чувак с лекториума приводил пример, разобранный в талмуде. Как к этому можно приобщиться? желательно без обрезания
мимо-гой
Аноним 14/06/15 Вск 03:09:46 #10 №275162 
>>275159
Ну технически это была фигуры речи, чтобы напомнить, что она здесь используется по существу.
А так, аксиома выбора в основном нужна либо, чтобы доказывать в максимальной общности утверждения, которые в большинстве здравых примеров можно доказать и без неё (хотя изредка ситуация и правда становится проблематичной), либо для теории кардинальных чисел. С другой стороны, она ведет к патологическим примерам. Если вместо неё взять одну из разумных аксиом (самая популярная здесь - это аксиома детерминированности), не совместимых с ней, то это ведёт к несколько другой математике (в разделась существенно использовавших аксиому выбора). Правда все это, конечно, исследовано пока весьма поверхностно, что явно говорит в пользу выбора для работающего математика.
Аноним 14/06/15 Вск 03:30:20 #11 №275165 
>>275162
>аксиома выбора в основном нужна либо, чтобы доказывать в максимальной общности утверждения
Вообще сложно разобраться по ходу дела, какими аксиомами ты пользовался во время доказательства. И даже доказывая утверждения не "максимальной общности" (что бы это ни значило), возможно интуитивно будешь прибегаешь к аксиоме выбора.
Всё же, чем конкретно тебя не устраивает аксиома выбора?
Аноним 14/06/15 Вск 03:49:41 #12 №275167 
>>275165
Я на всякий случай уточню, что я не столько математик, сколько логик и для меня не стоит вопроса выбора одной единственной "истинной"(чтобы это не значило), исходя из которой я и буду работать, аксиоматики.

Преимущества же отказа от аксиомы выбора в пользу аксиомы детерминированности состоят в том, что вместо некоторых патологических примеров доказываются их отрицания (например, для теоремы Банаха Тарского).

Касательно аксиом в доказательствах. Если немного попрактиковаться и следить за этим во время проведения доказательства, то в контроле использования, например, аксиомы выбора нет ничего сложного (проблемы здесь имеются скорее с "очевидными" аксиомами).
Аноним 14/06/15 Вск 06:02:47 #13 №275179 
>>275167
>не столько математик, сколько логик
Я помню тебя - ты тот кун, который пишет много о разном пространными фразами, но конкретно - практически ни о чём.

>например, для теоремы Банаха Тарского
Логических противоречий там нет, тебя как беспристрастного логика всё должно устраивать. Ну ладно, тогда как быть с тем фактом, что из множества натуральных чисел можно составить два равномощных ему множества, - утверждение сродни теоремы Банаха-Тарского, интуитивно непонятное, - его тоже можно считать "патологическим примером"? Да и в чем заключается патологичность?
Аноним 14/06/15 Вск 10:07:59 #14 №275195 
Вероятно, не тот тред, но в образоваче хуй ответят. Вот иду на матфак ВШЭ или мех-мат и в НМУ, но я не мат школьник занимаюсь этими делами 1.5 года, охватил лишь школьную программу и совсем немного олимпиадной математики. Так вот как поднять свой уровень знаний до выпускников 57ой
ну или хотя-бы что-бы на первых лекциях понимать материал.
Смотрел некоторые лекции первого курса НМУ ничего сложного, но приходиться постоянно гуглить и просматривать книги. Так вот посоветуйте подготовительной литературы которую можно освоить до конца лета. Сейчас пытаюсь читать всяких Зоричей и Кострикиных, но задачи решаю очень плохо, также думаю использовать литературу первого уровня отсюда http://hbpms.blogspot.ru/ но её овердохуя непонятно какую именно.
Аноним 14/06/15 Вск 11:17:40 #15 №275207 
>>275195
Ну 57 не все поголовно такие гении.
Изучай учебник Городенцева по алгебре/винберга, можно Зорича еще(но тут все не однозначно). Порешивай задачки из листков нмушных. Можно продолжать список, но на лето наверное тебе хватит этого. Можешь еще начать читать харламов/виро/.. Элементарная топология, там базовые вещи есть, которые надо знать(но много того чего и не обязательно, так что без фанатизма)
Возможно полезно заглянуть в программу вербицкого для матшкольников и не только
Ну и в нму ходи обязательно, полезная штука.
Аноним 14/06/15 Вск 11:20:44 #16 №275209 
>>275179
>как быть с тем фактом, что из множества натуральных чисел можно составить два равномощных ему множества
Никак. Это наблюдение связано с совершенно базовыми интуициями, касательно бесконечных множеств. Я не слышал ни о каких подходах к борьбе с ним, кроме полного отказа от бесконечности. Я, сколь-нибудь серьёзно, агитировать за такое не стану, да и сам не хочу загонять себя в подобные ограничения - это же страшно обеднит математику. Касательно патологичности, лично для меня здесь нет ничего контринтуитивного, в отличие от, скажем, возможности вполне-упорядочить вещественную прямую без возможности построить явного примера такого порядка (если по этому поду кто-то думает иначе, я не собираюсь их переубеждать).


Ситуация с аксиомой выбора другая. В силу некоторых результатов теории множеств, грубо говоря, верно следующее: если есть какая-то теорема, в формулировке которой фигурируют лишь счётные или счётно-заданные континуальные объекты (например гильбертовы пространства и непрерывные операторы в них) и нет произвольных подмножеств вторых, то если такая теорема доказуема в ZFC, то она доказуема и в ZF+DC+AD(акс. зависимого выбора и акс. детерминированности). Для чистой ZF это не верно, но тем не менее любой контрпример к этому с неизбежностью будет весьма изощрёнен. Кроме того, что теряется не много, отказ от аксиомы выбора в пользу аксиомы детерминированности даёт доступ к альтернативной математике. Например, то что все подмножества вещественной прямой измеримы по Лебегу во многих случаях будет большим удобством, чем возможность проводить построения по трансфинитной индукции вдоль, например, континуума.

И еще раз, у меня нет принципиальных претензий к аксиоме выбора, я лишь указываю на по-своему красивые альтернативы к ней.

ЛОГИЧЕСКИЙ РЕШАТЕЛЬ ЗАДАЧ Аноним 14/06/15 Вск 11:36:33 #17 №275214 
Есть кто из МАТИСа в МГУ? Интересует личность Подколзина А. С. и его логического решателя. Хуета/нет?
Аноним 14/06/15 Вск 12:03:54 #18 №275221 
>>275152
Тогда ℝ = ℚ×ℚ и, следовательно, ℝ счётно. Но ℝ несчётно. Следовательно, размерность не может быть счётной.
Аноним 14/06/15 Вск 12:14:03 #19 №275225 
>>275209
> все подмножества вещественной прямой измеримы по Лебегу во многих случаях будет большим удобством
а чем это удобно и что с этим можно делать?
Аноним 14/06/15 Вск 12:24:28 #20 №275228 
>>275225
Нет мороки с тем, чтобы доказывать, что какие-то функции и и множества измеримы - все измеримы по Лебегу. Упрощаются определения связанные с теорией меры т.к. опять же можно предполагать измеримость всех множеств. Разумеется ничего принципиального, но тем не менее.
Аноним 14/06/15 Вск 12:28:07 #21 №275229 
>>275228
И заодно нельзя доказать теорему Больцано-Коши (если непрерывная функция, определённая на вещественном интервале, принимает два значения, то она принимает и любое значение между ними). О-о-очень удобно.
Аноним 14/06/15 Вск 12:28:37 #22 №275230 
>>275158
Это же просто обратная задача к задаче из оп-поста, да?
Аноним 14/06/15 Вск 12:33:33 #23 №275232 
>>275229
Можно, конечно. Эти проблемы возникают в конструктивном анализе, где они загоняют себя в куда более жесткие рамки чем просто отказ от аксиомы выбора.
>>275230
Что ты имеешь в виду? Это задача про то, что для решения задачи из ОП-поста неизбежно нужна аксиома выбора.
Аноним 14/06/15 Вск 12:41:01 #24 №275237 
>>275232
Разве теорию R можно построить без аксиомы выбора?
Аноним 14/06/15 Вск 12:42:29 #25 №275238 
>>275232
Математика без аксиомы выбора это просто зашквар. Вся алгебра летит к чертям
Аноним 14/06/15 Вск 12:45:52 #26 №275243 
>>275238
Аксиома выбора - это как вера в боженьку - сам я не знаю как это сделать, но ведь есть боженька и он всемогущ.
Аноним 14/06/15 Вск 12:46:16 #27 №275244 
>>275237
Да. Фактически, любую классическую теорему анализа можно доказать без нее. Более того, она и не используется в стандартных курсах анализа.
Аноним 14/06/15 Вск 12:48:30 #28 №275245 
>>275238
Только на алгебраических структурах большой мощности. Для всех счётных и для всех естественных континуальных полная аксиома выбора на самом деле не нужна.
Аноним 14/06/15 Вск 12:52:53 #29 №275249 
>>275244
А как же доказать эквивалентность непрерывности R по Коши-Кантору, Гейне-Борелю и Больцано-Вейерштрассу? Там же везде выбор.
Аноним 14/06/15 Вск 12:55:08 #30 №275250 
>>275249
Ну запость/дай ссылку на какое-нибудь доказательство и покажи, где ты увидел использование аксиомы выбора.
Аноним 14/06/15 Вск 12:59:20 #31 №275254 
>>275245
Да, пожалуй, с естественными континуальными я погорячился, но со счетными все должно быть в порядке.
Аноним 14/06/15 Вск 13:00:52 #32 №275255 
Да всем похуй на эту вашу конструктивность и логику вообще.
Все нормально используют ZFC и радуются жизни.
Аноним 14/06/15 Вск 13:01:38 #33 №275257 
>>275254
Существование алгебраического замыкания как доказывать?
Ну обычно жизнь нетерова(там где я видел), так что да можно и без нее.
Аноним 14/06/15 Вск 13:17:22 #34 №275264 
>>275257
На счётных структурах вроде никаких проблем. Строим последовательность расширяющихся полей Fn. F0 стартовое поле. Для получения F(n+1) рассмотрим линейное упорядочивание многочленов от x над Fn по типу натуральных чисел. Далее поочередно расширяем Fn корнями этих многочленов. Их объединение и есть алгебраическое замыкание. Единственное что осталось проверить, что можно явно и детерминировано предъявлять эти упорядочивания многочленов, если дано некоторое упорядочивание поля F0. Здесь тоже ничего сложного. Для построения упорядочивания многочленов достаточно упорядочивание элементов поля. Для данного упорядочивания поля и неразложимого многочлена над ним легко упорядочить и поле разложения этого многочлена.
Аноним 14/06/15 Вск 13:45:48 #35 №275271 
>>275243
Математики и являются верующими, это даже многие математики сами говорят.
Аноним 14/06/15 Вск 13:52:29 #36 №275275 
Матиматико-куны,расскажите почему математики не выводят новых аксеом на основе эксперементов(почему все пытаются доказать), положим, проблема гольдбаха. Можно ли посчитать ее решенной на основе проверки ее истинности в , например, 2000 эксперементах(с разложением чисел), как в естественных науках.
Аноним 14/06/15 Вск 13:56:28 #37 №275277 
>>275271
Наверно верующие те, у которых философия плохо прокачена.
Аноним 14/06/15 Вск 14:02:26 #38 №275281 
>>275275
Зачем делать аксиомой то, что вполне вероятно в принципе можно доказать (или опровергнуть)? А так, если из гипотезы извлекаются интересные следствия, то просто говорят, что доказательство проводится в предположение верности этой гипотезы.
Аноним 14/06/15 Вск 14:03:28 #39 №275283 
>>275278
Не вижу проблемы.Если на 100000 чисел выявляется какая-либо закономерность, то в принципе можно считать ее аксиомой( почему нет).
Аноним 14/06/15 Вск 14:07:43 #40 №275286 
>>275275
Потому что математика и занимается тем, что выводит на основе данных аксиом теоремы. Тем самым давая возможность не искать эмпирмических подтверждений теорем.
Аноним 14/06/15 Вск 14:08:04 #41 №275287 
>>275283
Вот занятный пример для тебя
http://mathoverflow.net/a/27942
Аноним 14/06/15 Вск 14:41:21 #42 №275308 
>>275286
А откуда берутся аксиомы?
Аноним 14/06/15 Вск 14:43:31 #43 №275312 
>>275308
Математики в основном заимствуют их из других наук, лишь иногда обобщают свои собственные идеи. Если выбрать аксиомы от балды, то плодотворной теории не получается.
Аноним 14/06/15 Вск 14:50:10 #44 №275325 
В алгебре постоянно дрочат на степенные уравнения, типа квадратных или третьей степени. Я правильно понял, что причиной этому является, что любая f(x), являющаяся произвольной комбинацией операций и элементов из поля, сводима к полиному? Т.е. имея сложение и умножения, и комбинируя их как вздумается, мы всегда получим степенное уравнение?
Аноним 14/06/15 Вск 14:52:05 #45 №275326 
>>275308
Интуитивно очевидные принципы, касающиеся предмета рассмотрения. Так было ещё у Евклида.
Аноним 14/06/15 Вск 14:53:16 #46 №275329 
>>275326
Интересно, можно ли придумать дурацкую геометрию с йоба аксиомами? Не думаю, тогда интуицию напрочь отобъет.
Аноним 14/06/15 Вск 14:55:26 #47 №275334 
>>275329
Кроме аксиом, важны ещё и определения (то есть объекты), и факты об объектах. Если тебе удастся придумать аксиомы, изобрести кучу йоба-объектов и доказать о них много содержательных теорем, то я признаю тебя гением.
Аноним 14/06/15 Вск 14:56:08 #48 №275336 
>>275331
Но ведь даже в топовых алгебраических геометриях и йоба теорий чисел исследуются те же самые степенные формы.

Алсо, неразрешимость уравнения над полем есть существование набора композиций, выходящих за пределы поля?

не бейте, лучше обоссыте, я школьник
Аноним 14/06/15 Вск 14:58:04 #49 №275340 
>>275334
Если бы меня заставили под дулом пистолета сделать ставку на то, возможно ли такое, я бы сказал, что нет.
Аноним 14/06/15 Вск 14:59:34 #50 №275344 
>>275338
Но ведь там как раз таки не нарушается интуиция. Пятая аксиома неинтуитивная. По-моему параллельная прямая по определению та, которая не пересекается.
Аноним 14/06/15 Вск 15:01:36 #51 №275348 
>>275243
Так можно про всю математику сказать.
Аноним 14/06/15 Вск 15:02:19 #52 №275349 
>>275338
Евклид ввел понятие параллельной прямой. Это прямая не пересекающаяся с данной.
Лобачевский построил геометрию без этого объекта.
Риман построил геометрию с бесконечным количеством таких прямых.
Аноним 14/06/15 Вск 15:03:10 #53 №275350 
>>275348
Существует мнения, что математикам похуй на аксиомы, они выводят теории.
Аноним 14/06/15 Вск 15:05:59 #54 №275352 
>>275349
Кажется я обосрался. Перепутал Римана с Лобачевским.
Аноним 14/06/15 Вск 15:13:10 #55 №275358 
>>275350
Аксиомы и определения — самое главное в математике.
Аноним 14/06/15 Вск 15:17:19 #56 №275360 
Математика, по факту, всего лишь логика, с добавлением свойств чисел.
Аноним 14/06/15 Вск 15:19:10 #57 №275361 
Что за сборище петухов
Проверять аксиомы
Интуитивность
вы ебанутые сука
Аноним 14/06/15 Вск 15:26:23 #58 №275368 
Аксиомы на эмпирике построены. 2+2=4,аксиома, потому что сколько(и какими методами) ты бы не складывал, все ровно это получится.если 10000 экспериметов провести, все ровнотак и будет.
Аноним 14/06/15 Вск 15:28:47 #59 №275369 
Короткий IQ тест.
Любишь основания?
Нет. IQ=27 (С головой problemes.)
Да. IQ=289 (Больше, чем у Эйнштейна.)
Аноним 14/06/15 Вск 15:33:44 #60 №275376 
>>275369
Эксперимент и статистика являются основанием?
Аноним 14/06/15 Вск 15:35:41 #61 №275377 
>>275376
Основанием может быть все что угодно. Важно то как ты к нему относишься.
Аноним 14/06/15 Вск 15:38:35 #62 №275382 
>>275369
>любишь основания?
NaOH-обожаю.
Аноним 14/06/15 Вск 15:50:03 #63 №275405 
>>275377
Я люблю его!
Аноним 14/06/15 Вск 16:12:27 #64 №275442 
Тут кто-нибудь учится в СПб в ИТМО/Политехе/ЛЭТИ? Можете что-нибудь сказать про них? Я хочу на Матмех, но не уверен, что пройду, поэтому рассматриваю эти варианты.
Аноним 14/06/15 Вск 16:23:35 #65 №275459 
а этот чувак жив или как?
https://vk.com/neugodoff
Аноним 14/06/15 Вск 16:29:30 #66 №275469 
14342885709600.jpg
Это ведь тот, который хотел переправиться в США на лодке? Если он выплыл в океан на этом, то думаю уже нет.
Аноним 14/06/15 Вск 17:48:12 #67 №275533 
14342932924230.png
>>275209
До теоремы Банаха-Тарского доходят такими же "базовыми интуициями", как и в случае с удвоением множества натуральных чисел. Есть сотни эквивалентных аксиоме выбора утверждений, и если вполне-упорядочение континуума кажется тебе "контринтуитивным", то возможно какие-то другие покажутся очевидными. Например, утверждения о том, что между двумя любыми множествами есть инъекция или, что произведение непустых множеств не пусто, кажутся мне достаточно интуитивными.

>счётно-заданные континуальные объекты
Что значит счётно-заданные континуальные объекты?

Не могу сообразить почему аксиома детерминированности несовместима с аксиомой выбора. Множество стратегий, - что для первого, что для второго игрока, - может быть континуальным, им обоим приходится выбирать выигрышную стратегию из этого множества, то есть, уже, как я понимаю, нужно использовать аксиому выбора. Это что касается словесной формулировки. С формальной - всё яснее. Там выигрышная стратегия формируется как бы по ходу игры и выбирать приходится только элементы из счетного множества S, а для этого аксиома выбора уже не нужна. Но, опять же, из формального определения непонятно в чем несовместимость.
Аноним 14/06/15 Вск 17:56:12 #68 №275545 
>>275533
> или, что произведение непустых множеств не пусто
пиздец, неужели кто-то занимается доказательством таких вещей?
Аноним 14/06/15 Вск 18:12:00 #69 №275560 
>>275545
Это же аксиома. Но есть такие - кому подобные вещи кажутся не очевидными и интуитивно непонятными.
Аноним 14/06/15 Вск 18:15:25 #70 №275562 
>>275545
А потом хуяк - и твоя теория противоречива.
Аноним 14/06/15 Вск 18:32:15 #71 №275571 
>>275146
Как определить, что бесконечное, возможно несчётное, подмножество векторов линейно независимо?
Аноним 14/06/15 Вск 18:53:55 #72 №275588 
>>275545
Ну это не для пары множеств, а для произведений по произвольных семейств.
>>275533
>Например, утверждения о том, что между двумя любыми множествами есть инъекция или, что произведение непустых множеств не пусто, кажутся мне достаточно интуитивными.
Мне нет.
>Что значит счётно-заданные континуальные объекты?
Я несколько сомневался стоит ли об этом упоминать, когда писал тот пост - дело несколько запутанное. Если быть формально точным, то речь шла о том, что если утверждение в языке арифметики второго порядка доказуемо в ZFC, то оно доказуемо и в той теории. О счётных структурах в этом языке говорить совсем легко - просто рассматриваются структуры, носителем которых являются натуральные числа (а каждая счётная изоморфна такой). Но тем не менее об многих континуальных тоже можно. Например, о полных метрические сепарабельные пространствах. Такое пространство задаётся счётным всюду плотным множеством и функцией расстояния, а это можно закодировать в одном множестве натуральных чисел. Точки в таких пространствах тоже могут кодироваться множествами натуральных чисел. Таким образом возможны кванторы и по пространствам и по точкам в них. В таком же духе можно много что закодировать.

Построить недетерменированное множество с помощью аксиомы выбора просто. Стратегия игрока это функция из мн-ва конечных последовательностей натуральных чисел в множество натуральных чисел. Рассмотрим дизъюнктивное объединение стратегий обоих игроков, оно очевидно континуально. Вполнеупорядочим его с наименьшим возможным порядковым типом. Теперь трансфинитной индукцией по нему строим два семейства расширяющихся множеств Ga и Ea (a - пробегает элементы дизъюнктного объединения). Мы будем строть их так, что каждые Ea и Ga не пересекаются, на каждом шаге оба менее чем континуальны и нашим недетерминированным множеством будет объединение всех Ga. Рассматривая очередную стратегию a мы замечаем, что при всевозможных стратегиях другого игрока возможно континуально-много различных исходов и тем самым мы сможем гарантировать проигрышность a, добавив либо в Ea, либо Ga один исход из этого семейства, который еще не лежит ни в Ea ни в Ga.

Аноним 14/06/15 Вск 19:04:28 #73 №275599 
>>275588
Приведи пример множеств, между которыми нет инъекции.
Аноним 14/06/15 Вск 19:08:59 #74 №275605 
>>275599
Например, мне совершенно не ясно почему должна быть инъекция между континуумом и первым несчётным ординалом.
sageАноним 14/06/15 Вск 19:14:46 #75 №275613 
Теория множеств - скучно. Уебывайте.
sageАноним 14/06/15 Вск 19:15:35 #76 №275614 
>>275588
>либо Ga
>ga
Вот и уебывай в /ga/ со своей теорией множеств.
Аноним 14/06/15 Вск 19:17:26 #77 №275618 
>>275614
Лел, оригинальный ход мысли.
Аноним 14/06/15 Вск 19:22:20 #78 №275624 
>>275605
Потому что если счётное количество раз извлечь из континуума счётное множество, в континууме всё равно останутся точки.
Аноним 14/06/15 Вск 19:23:11 #79 №275626 
>>275624
И?
Аноним 14/06/15 Вск 19:28:38 #80 №275633 
>>275626
Из этого следует, что инъекция быть должна.
Аноним 14/06/15 Вск 19:30:50 #81 №275638 
>>275633
Нет. Я так понял, что ты хочешь использовать то, что первый несчётный ординал - это счётное объединение счетных множеств. И откуда ты это взял?
Аноним 14/06/15 Вск 19:41:08 #82 №275651 
>>275624
>>275633
Ладно, кажется я понял, что ты имел ввиду. По сути ты апеллируешь к трансфинитной рекурсии с выбором вдоль ординала. Лично мне это не кажется столь уж интуитивным.
Аноним 14/06/15 Вск 19:44:30 #83 №275654 
Как пофиксить OCSP-ответ ещё не действителен (содержит дату из будущего). в лисе на википедии?
Аноним 14/06/15 Вск 19:46:22 #84 №275655 
>>275651
А я считаю, что это не просто интуитивно, а ещё и очевидно.
Аноним 14/06/15 Вск 19:47:35 #85 №275658 
>>275588
>Стратегия игрока это функция из мн-ва конечных последовательностей натуральных чисел в множество натуральных чисел.
А как с помощью такой функции из конечных последовательностей мы дойдем до конца omega-игры - количество ходов в которой счетно бесконечно?
Ну допустим на своём первом ходу игрок I получает ряд (0), на втором (0,1,2), на третьем - (0,1,2,4,5). Его стратегия, к примеру, выигрышная, всё нормально. В итоге имеем, через время, какой-то ряд (0,1,2,4,5,6,....n). Но это не конец omega-игры, и конца мы не получим, потому что длинны последовательностей ограничены конечным числом (хотя в общем-то брать их можно счётно-бесконечно много раз, но до омеги мы так и не доберемся или таки доберемся?).
Аноним 14/06/15 Вск 19:55:39 #86 №275663 
>>275658
Игра происходит примерно так:
*. есть две стратегии (т.е. ф-ии из мн-ва конечных последовательностей) f и g;
0. стартуем c пустой последовательности a0=[];
1. дальше a1=[f(a0)]=[f([])]
2. дальше a2=[f(a0),g(a1)]=[f([]),g([f([])])]
.
.
.
2n+1. a(2n+1)=[f(a0),g(a1),..,f(a(2n))]
.
.
.

Результат это последовательность [f(a0),g(a1),..,f(a(2n)),g(a(2n+1)),..].
Аноним 14/06/15 Вск 19:58:36 #87 №275667 

>>275207
спасибо тебе анон
Аноним 14/06/15 Вск 21:24:59 #88 №275745 
>>275663
Понятно, что здесь постулируется, что один из игроков должен выиграть, но мне кажется совсем не очевидным, что игра завершится. С последней последовательностью все не так ясно. Хоть и длина её выходит omega, но все её члены конечные и в то же время этими членами как бы определяется длина последовательности.
Аноним 14/06/15 Вск 22:06:10 #89 №275753 
>>275161
Что, ни одного еврея здесь нет?
Аноним 14/06/15 Вск 22:07:04 #90 №275754 
>>275753
У меня дед
Аноним 14/06/15 Вск 22:21:01 #91 №275761 
>>275753
>Как к этому можно приобщиться?
Читай талмуд, ешь мацу, соблюдай шаббат, празднуй хануку - со временем глубокое понимание математики придет.
Аноним 15/06/15 Пнд 00:20:41 #92 №275769 
господа, как решить x^log[2,x+2]=256?
Аноним 15/06/15 Пнд 01:35:04 #93 №275778 
14343213040550.png
>>275769
Аноним 15/06/15 Пнд 01:59:17 #94 №275781 
>>275778
Тролль что ли? 254^8 = 256, вообще охуеть.
Аноним 15/06/15 Пнд 02:22:48 #95 №275783 
>>275777
обисрався, дякую за допомогу
>>275778
обисрався
Аноним 15/06/15 Пнд 02:45:50 #96 №275785 
Господа, помогите убогому с задачей.

Найдите всевозможные значения параметра а, такие что система уравнений

Система уравнений:
|x^2+y^2-16|+8x-8y+16=0
y=ax+2

имеет более 2х решений.

Не могу привести подобные из-за модуля (понимаю,что решать через дискриминант.
Аноним 15/06/15 Пнд 03:19:20 #97 №275788 
>>275769
Нерешаемо. Можно только сказать, что корень один и он примерно равен 6,2.

>>275785
Как насчет раскрыть модуль по определению?

Аноним 15/06/15 Пнд 03:21:30 #98 №275789 
>>275788
А можешь объяснить, кстати, почему нерешаемо. Ответа кстати два(4 и 1\16)
Аноним 15/06/15 Пнд 03:38:37 #99 №275790 
>>275788
так я не могу сказать какой знак имеет выражение, заключенное в модуль.
Аноним 15/06/15 Пнд 05:05:09 #100 №275792 
>>275790
Ты что, дурачек?
Аноним 15/06/15 Пнд 05:25:41 #101 №275798 
>>275789
>>Ответа кстати два(4 и 1\16)
>>x^log[2,x+2]=256

Васян, ты б "ответы" свои подставил в уравнение, прежде чем в лужу пердеть.
Вот 4, например.
4^log(2; 4+2)=2^log(2; 6^2)=36
Аноним 15/06/15 Пнд 05:30:31 #102 №275799 
>>275790
>>так я не могу сказать какой знак имеет выражение, заключенное в модуль.

Попробуй обозначить на плоскости XY все пары (x y), при которых выражение под модулем положительно, затем построй линию f(xy)=0 для этого случая. То же для отрицательного.

Я не уверен, что это оптимальный способ решить твою задачу, но решение ты получишь наверняка.
Аноним 15/06/15 Пнд 09:28:02 #103 №275808 
>>275146
Цепью называется линейно упорядоченное подмножество частично упорядоченного множества.
Лемма Цорна. Если в частично упорядоченном множестве любая цепь имеет какую-то верхнюю грань, то во множестве существует хотя бы один максимальный элемент.

Пусть есть какое-то семейство множеств {Ai}, индексированное множеством натуральных чисел. Назовём это семейство цепью множеств, если A1⊂A2⊂A3⊂...

Лемма 1. Пусть для некоторых i и j Ai и Aj - элементы цепи множеств. Тогда Ai⊂Aj или Aj⊂Ai.

Лемма 2. Объединение любого конечного подсемейства цепи множеств есть элемент этой цепи.
Доказательство по индукции. Пусть A и B - два элемента цепи. Тогда, по первой лемме, A⊂B или B⊂A. То есть A∪B = A или A∪B = B.

Будем рассматривать только нетривиальные векторные пространства, то есть пространства, в которых существует вектор, отличный от нулевого.
Пусть V - векторное пространство над полем P.
Пусть S - конечное семейство векторов, в нём n элементов.
S называется линейно зависимым, если существуют такие скаляры a1, ..., an, среди которых хотя бы один не равен нулю, что a1s1 + a2s2+...+ansn = 0.
S называется линейно независимым, если оно не является линейно зависимым.
Пусть S - бесконечное семейство векторов. Оно называется линейно независимым, если любое его конечное подсемейство линейно независимо.

Лемма 3. Объединение цепи множеств {Ai}, элементами которой являются линейно независимые семейства, есть линейно независимое семейство.
Доказательство. Пусть объединение не является линейно независимым. Тогда в нём есть конечное подсемейство, не являющееся линейно независимым. Пусть оно состоит из векторов x1, ... , xn. Каждый xi принадлежит какому-то элементу Ai. Объединение этих элементов, по лемме 2, есть элемент цепи, обозначим его A. Значит, A не является линейно независимым, что противоречит условию.

Если линейно независимое семейство векторов не является собственным подсемейством какого-то линейно независимого семейства, то оно называется базисом.

Теорема 1. В любом векторном пространстве VP существует базис.
Доказательство. Поскольку V нетривиально, в нём есть вектор v, не равный нулю. Семейство v1 линейно независимо. Пусть M - множество всех линейно независимых семейств V. M не пусто. Упорядочим M по включению, то есть a≺b, если a⊂b. Пусть a1 ≺ a2, ... - цепь. Положим a = ∪ai. По лемме 3, a ∈ M. Значит, любая цепь элементов M мажорируется каким-то элементом M. Значит, в M есть максимальный элемент, обозначим его m. По определению, m - базис.

Теорема 2. Любой вектор может быть представлен в виде линейной комбинации конечного подсемейства базиса (иначе говоря, разложен по базису).
Доказательство. Заметим, что если a есть линейная комбинация векторов xi, то семейство <xi, a> линейно зависимо. Пусть m - базис. Предположим, что a не может быть разложен по базису. Тогда <m, a> - линейно независимое семейство. Что противоречит определению базиса.

Лемма 4. ℝ есть векторное пространство над полем ℚ.
Следствие. В ℝ есть базис.

Теорема 2. Базис ℝ =: m имеет мощность континуума.
Доказательство. ℝ равномощно множеству всех конечных подмножеств m. Множество всех конечных подмножеств m равномощно m.
Аноним 15/06/15 Пнд 10:36:56 #104 №275841 
>>275808
А где доказательство линейной независимости в последней теореме?
Аноним 15/06/15 Пнд 11:03:06 #105 №275844 
>>275841
Нафига оно там?
Аноним 15/06/15 Пнд 11:28:43 #106 №275856 
>>275844
Чтобы доказать, что базис имеет мощность континуума, нужно показать что есть континуальное количество линейно независимых векторов. В общем-то, это самая технически проблемная часть доказательства.
Аноним 15/06/15 Пнд 11:53:13 #107 №275866 
>>275856
m - базис.
С одной стороны, всякому элементу ℝ соответствует конечная линейная комбинация. Это соответствие инъективно.
С другой стороны, всякая одноэлементная линейная комбинация есть элемент ℝ.
Значит, по теореме Кантора-Бернштейна ℝ равномощно множеству всех конечных линейных комбинаций элементов m.
Линейная комбинация длины n - это элемент множества [1..n]×ℚ×m.
Предположим, что m конечно или счётно. Тогда [1..n]×ℚ×m счётно.
Тогда множество всех конечных линейных комбинаций счётно.
Тогда ℝ счётно.
Значит, m несчётно.
Тогда [1..n]×ℚ×m равномощно m.
Откуда множество всех конечных линейных комбинаций равномощно m.
То есть ℝ равномощно m.
Аноним 15/06/15 Пнд 12:15:40 #108 №275874 
Простите что отвлекаю от гомотопий и прочих теорий групп, но я в тупике.
Накидайте литературы и мануалов по решению линейных дифуров с переменными коэффициентами.
Уже 3ю пересдачу не могу найти частное решение.
Вольфрам выдаёт ерунду
(x^3-2x^2)y''+4x^2y'-6xy=0
Аноним 15/06/15 Пнд 12:17:22 #109 №275875 
>>275874
Я знаю гомотопии, но я не знаю эти твои дифуры. Сорри.
Аноним 15/06/15 Пнд 12:20:48 #110 №275877 
>>275875
Это понятно. Но бы тоже хотел знать гомотопий, но их преподают на 3м курсе
Аноним 15/06/15 Пнд 12:30:39 #111 №275881 

>>275808
Не понимаю, зачем ты столько расписал. Очевидно, что мощность базиса не больше континуума. Но и меньше быть не может, согласно континуум-гипотезе, т.к Q-линейные комбинации счётного базиса дадут счётное множество.
Аноним 15/06/15 Пнд 12:39:01 #112 №275884 
>>275881
>согласно континуум-гипотезе
Просто я ей не пользовался.
Аноним 15/06/15 Пнд 12:39:06 #113 №275885 
>>275866
Sounds about right.
Аноним 15/06/15 Пнд 12:41:54 #114 №275886 
>>275881
А что континуум-гипотеза уже стала аксиомой?
Аноним 15/06/15 Пнд 12:44:33 #115 №275888 
>>275881
>Q-линейные комбинации счётного базиса дадут счётное множество
Тут континуум-гипотеза не нужна, тут аксиома выбора.
Аноним 15/06/15 Пнд 12:47:20 #116 №275889 
>>275888
В смысле, базис несчётен и не более чем континуален. Промежуточных мощностей между континуумом и нулевым алефом нет, значит, базис континуален.
Аноним 15/06/15 Пнд 12:57:09 #117 №275895 
>>275894
Ты какой-то ебанутый.
Аноним 15/06/15 Пнд 13:15:23 #118 №275913 
>>275889
Мы тут когда говорим континуум, подразумеваем мощность R. А то, что мощность R (равная 2^omega) строго больше омеги следует из теоремы Кантора. Континуум-гипотеза не нужна.
А вот уже чему равно 2^omega - хуй знает без континуум-гипотезы. Но нам это и не нужно в данном случае.
Аноним 15/06/15 Пнд 13:16:00 #119 №275914 

>>275884
На самом деле там она не нужна, т.к даже если континуум гипотеза неверна и есть А такое что |А| меньше континуума и больше счетного то |A×Q|=|A| и континуума с таким базисом мы не получим. Так что нужна только аксиома выбора, но без неё уж никуда.
Аноним 15/06/15 Пнд 13:20:16 #120 №275915 
>>275897
ОП-пик - математик, зачем ему нести такую хуйню?
https://en.wikipedia.org/wiki/Matthew_Foreman
Аноним 15/06/15 Пнд 13:28:46 #121 №275923 
>>275914
>|A×Q|=|A|
И что?
Аноним 15/06/15 Пнд 15:48:42 #122 №276022 
14343725224260.jpg
Все уже заценили новую работу Ромы?
http://arxiv.org/abs/1506.00952
Аноним 15/06/15 Пнд 16:19:10 #123 №276030 
>>275874
Читай Камке справочник по обыкновенным дифурам. После сокращения на х там выйдет что-то вроде гипергеометрического уравнения. В виде бесконечного ряда получить решение довольно просто - идеоогия как для Бесселя. Есть ли более просто - хз. Лень возиться.
Аноним 15/06/15 Пнд 16:32:30 #124 №276034 
>>275915
Интересный факт. Полазил по графе Doctoral advisor. Оказывается ОП-пик наследник Германа Вейля, от него к Гильберту, Дирихле и так через кучу знаменитых математиков, включая Иоганна Бернулли. Огромное количество выдающихся математиков были учениками друг друга. Самая настоящая династия.
Аноним 15/06/15 Пнд 16:43:34 #125 №276035 
Сегодня читал Зорича и встретил фразу, которая меня немного загнала. Примерное содержание: «Если X, Y — мн-ва, определённые в c), то либо ∃ min X, либо ∃ max Y.» Речь шла о теореме Дедекинда. Меня смущает союз либо… либо. Вроде понимаю, что они не взаимоисключают друг друга (да и в самой задаче они могут выполняться одновременно), но как-то это странно звучит. Есть ли среди математиков определённая договорённость в подобных случаях?
Аноним 15/06/15 Пнд 16:44:04 #126 №276036 
Анон, как учить алгебру, например? Вот я читаю учебник Винберга. Запомнить и понять все эти определения трудно, потому что без использования их на практике они быстро забываются. Каких-то теоретических упражнений там нет. И как же выучить алгебру?
Аноним 15/06/15 Пнд 16:48:30 #127 №276037 
>>276035
Нет, договорённости нет. Ни Академия наук, ни ММО не принимали рекомендаций по этому поводу.
На английском проще, там есть союзы either и neither. Рекомендую тебе считать, что либо, ... , либо соответствует either.
Аноним 15/06/15 Пнд 17:06:15 #128 №276042 
>>276037
либо ... либо ... = either ... or ...
просто either = любой/каждый/оба
https://proofwiki.org/wiki/Dedekind's_Theorem
"It is not possible that both of the above can happen."©
в данном случае именно "либо либо", так что все правильно переведено.
Аноним 15/06/15 Пнд 17:20:23 #129 №276044 
Анон, подскажи, есть ли какое-нибудь адекватное представление интеграла от непрерывной или хотя бы абсолютно непрерывной функции по сингулярной функции ограниченной вариации? Гугл, как я ни искал, выдает сингулярные интегралы, но, насколько я понимаю, они тут не при чем, или я не прав?
Аноним 15/06/15 Пнд 17:47:16 #130 №276052 
>>276042
Эм, но ведь написано: «α may be an element of either L or R.» И на or дана ссылка на дизъюнкцию.

>"It is not possible that both of the above can happen."
Но здесь же речь идёт об общем случае, когда множества могут иметь скачок. В Зориче на мн-ва X и Y накладывалось такое свойство: X ∪ Y = ℝ, и ∀ x ≤ ∀ y. Отсюда я доказал, что sup X = inf Y. Разве не может получиться так, что максимальный элемент будет равен минимальному? К примеру, два множества ]−∞; 1] и [1; +∞[. Вроде всё нормально. В чём прокол?
Аноним 15/06/15 Пнд 17:48:39 #131 №276054 
>>276030
Спасибо.
Походу вольфрам не ошибается и ответ действительно в виде ряда
Аноним 15/06/15 Пнд 17:51:13 #132 №276055 
14343798732240.jpg
>>276035
С хуя ли? Если бы стоял значок дизъюнкции, тогда да, утверждения не взаимоисключали бы друг друга.
А так либо… либо, в обычном языке, это выбор их двух альтернатив.
Аноним 15/06/15 Пнд 18:06:37 #133 №276067 
14343807972880.jpg
Аноны, помогите. Задание с ЕГЭ. Вроде решил, но неправильно. Помогите
Аноним 15/06/15 Пнд 18:12:50 #134 №276075 
14343811701540.jpg
>>276052
>>276042
Короч, то ли я обосрался, то ли Зорич. Здесь идёт речь о сечении. А у сечения в свою очередь есть свои свойства, одно из них: каждое рациональное число попадает в одно, и только в одно из множеств сечения. Это из Фихетнгольца. В Википедии написано по-другому: во-первых, стоит строгое неравенство, так что ∀ x < ∀ y, и написано, что нижний класс сечения не имеет максимального элемента. В Зориче ни слова про сечение не идёт. Я запутался, аноны. Что не так?
Аноним 15/06/15 Пнд 18:26:38 #135 №276088 
>>276067
Ты "x" пишешь по-уебански, не буду тебе помогать.
Аноним 15/06/15 Пнд 18:31:12 #136 №276095 
14343822722980.jpg
>>276092
>полуокружностебогов
Аноним 15/06/15 Пнд 18:40:34 #137 №276103 
>>276075
В первой половине прошлого века порядок определяли как отношение < со следующими свойствами.
1. Антирефлексивно, то есть для любого a неверно, что a<a
2. Асимметрично, то есть если a<b, то неверно, что b<a
3. Транзитивно, то есть если a<b и b<c, то a<c.

Начиная со второй половины прошлого века порядок определяют как отношение ≤ со следующими свойствами.
1. Рефлексивно, то есть для любого a верно, что a≤a
2. Антисимметрично, то есть если a≤b и b≤a, то a=b
3. Транзитивно, то есть если a≤b и b≤c, то a≤c.

Понятие "дедекиндово сечение" существенно зависит от того, какая версия порядка используется. То есть существуют, по сути, две версии этого понятия. Одна версия более старая, она изложена у Фихтенгольца и основана на отношении <. Другая версия более новая, она изложена у Зорича и основана на отношении ≤.

Тебе следует определиться, какую версию понятия "дедекиндово сечение" ты познаёшь. Если ты хочешь познать обе версии, то тебе не следует смешивать источники.
Аноним 15/06/15 Пнд 18:50:22 #138 №276107 
>>276103
Спасибо за ответ. Я уже и сам понял, что здесь всё дело в этом отношении. Ну вот смотри: ]−∞; 1] и [1; +∞[ — два мн-ва, удовлетворяющие свойствам на пике (>>276075)
, при этом оба имеют мажоранту/ миноранту соответственно. Или же я что-то не учитываю?
Аноним 15/06/15 Пнд 18:51:33 #139 №276108 
>>276107
>свойствам на пике
С пункта b).
Аноним 15/06/15 Пнд 18:56:22 #140 №276110 
>>276107
>]−∞; 1] и [1; +∞[
Да, они удовлетворяют свойствам.
Аноним 15/06/15 Пнд 19:01:10 #141 №276111 
>>276107
У ]−∞; 1] есть мажоранта, но нет миноранты.
У [1; +∞[ есть миноранта, но нет мажоранты.
Аноним 15/06/15 Пнд 19:03:34 #142 №276113 
>>276075
У Рудина почитай, про сечения норм поясняет. А насчёт строго и нестрогого неравенства: если сечение определяется рациональным числом - то неравенство нестрогое. А если сечение определяется числом, скажем квадрат которого равен 2, то неравенство строгое. Это и есть вещественный числа.
Аноним 15/06/15 Пнд 19:06:18 #143 №276114 
>>276110
Тогда получается, что у нас есть и миноранта, и мажоранта. Соответственно, союз либо… либо не взаимоисключающий. Но в теореме Дедекинад… А что в ней?.. А в ней возможен только один вариант из двух, то есть более строгое условие, которое доказать не получится, если только не взять непересекающиеся множества. Почему Зорич допускает, что один и тот же элемент будет принадлежать обоим множествам? Ведь если бы он сделал строгое неравенство, то никаких трудностей в доказательстве. Короче, я не понял Зорича в этом месте.
>>276111
Да, я это и написал.
>мажоранту/ миноранту соответственно
Аноним 15/06/15 Пнд 19:10:12 #144 №276117 
>>276099
Полуокружностебог в своей жизни не написал ни одной лекции по линейной алгебре.
Написание икса крестиком занимает меньше времени и в целом выражение содержащее икс получается читабельнее. А твои полукруги при быстром письме превращаются в вырвиглазное уебанство.
Аноним 15/06/15 Пнд 19:11:52 #145 №276118 
>>276114
>А в ней возможен только один вариант из двух
Почему?
Аноним 15/06/15 Пнд 19:19:05 #146 №276121 
>>276118
Такова формулировка в Фихтенгольце, к примеру. Да и здесь (https://proofwiki.org/wiki/Dedekind%27s_Theorem) тоже самое.
Доказательство идёт через сечения (а для них x < y), поэтому всё нормально доказывается. Зорич же предлагает доказать эту теорему, формулировку которой он даже толком не расписал, через множества, для которых x ≤ y, то есть с более слабым условием. Соответственно, доказать я могу только теорему с более слабой формулировкой, когда возможны оба случая одновременно. Так ли это?
Аноним 15/06/15 Пнд 19:27:19 #147 №276125 
В каком направлении движется современная математика? Точнее, какие области наиболее активно развиваются?

Я бы посмотрел и проанализировал последние конференции, но не знаю (пока) английского.

Аноним 15/06/15 Пнд 19:29:23 #148 №276126 
>>276121
У Фихтенгольца другая теорема. Зорич предлагает доказать не её, а то, что он пометил как теорема Дедекинда. То есть доказать нужно вот что.

Теорема Дедекинда. Пусть X ⊂ ℝ, Y ⊂ ℝ, X и Y не пусты и X∪Y = ℝ. Пусть для любого x из X и y из Y верно, что x≤y. Тогда верно по меньшей мере одно из двух утверждений. 1) существует max X; 2) существует min Y.
Аноним 15/06/15 Пнд 19:30:25 #149 №276127 
>>276125
imperium.lenin.ru/~verbit/MATH/programma.html
Аноним 15/06/15 Пнд 19:33:24 #150 №276130 
>>276127
Знаком. Ищу другие мнения.
Аноним 15/06/15 Пнд 19:38:11 #151 №276135 
>>276126
Ну вот и я про то же. Получается, что у него всё-таки союз либо… либо не разделяющий, а допускающий оба варианта сразу. Вот из-за этого у меня и пошло сомнение, ибо для меня либо… либо означает что-то одно из двух, но никак не два сразу. Собственно, с этого поста всё началось (>>276036). Вот из-за этого я и загнался. Как вообще я должен был догадаться о том, что у него либо… либо — это дизъюнкция, кроме контрпримера? Аноны выше ответили, что либо… либо — это either, то есть одно из двух. Ну и кто виноват во всём этом? Глупый я? Некошерность русского языка? Распиздяйство Зорича? Или просто неопределённость в конкретных выражениях?
Аноним 15/06/15 Пнд 19:39:28 #152 №276138 
>>276135
>с этого поста всё началось (>>276036)
С этого: >>276035
fix
Аноним 15/06/15 Пнд 19:40:01 #153 №276140 
>>276130
http://old.computerra.ru/offline/2008/719/347100/
Аноним 15/06/15 Пнд 19:40:34 #154 №276142 
>>276135
>Некошерность русского языка? Распиздяйство Зорича?
this
Аноним 15/06/15 Пнд 19:50:42 #155 №276146 
>>276130
А иначе никак, хочешь знать что происходит в современной математике, придется учить английский. Читай описания конференций, попутно и язык подтянешь
http://www.ams.org/meetings/calendar/mathcal
Аноним 15/06/15 Пнд 20:36:34 #156 №276168 
A domain, or integral domain, is a ring (with at least two elements) in which the cancellation law holds

Что здесь означает слово domain? Только область определения? Или кольцо целостности?
Аноним 15/06/15 Пнд 20:58:49 #157 №276181 
>>276168
Ну ёб трою мать, это же определение. Там дальше написано, что в твоём случае подразумевается под domain.
Аноним 15/06/15 Пнд 21:07:16 #158 №276187 
>>276181
Ну я вижу, что дано определение кольца целостности, но меня смутило, что domain это еще и область определения. Ебаные америкосы с их убогим языком без синонимов. У них даже мат убогий и нетворческий.
Аноним 15/06/15 Пнд 21:14:17 #159 №276191 
>>275146
Континуум. Задача изи.
Аноним 15/06/15 Пнд 21:50:22 #160 №276233 
>>275146
О, видал недавно этого математика в чатрулети. паржали с пацонами!)
Аноним 15/06/15 Пнд 22:36:48 #161 №276277 
>>276146
Так и есть, анон. Три раза в неделю сажусь и учу инглиш. Приятно замечать, что математика качает мозг. Быстро получается схватывать правила
>>276140
Ок, почитаю.


Аноним 15/06/15 Пнд 23:21:45 #162 №276287 
An element a in a ring R is irreducible if it is not a unit or zero, and for any factorization a = bc, b,c 2 R, either b or c is a unit

В каком значении здесь используется слово unit?
>if it is not a unit or zero
Если этот элемент не ноль и не единица? Или "единица" в смысле "обратимый элемент"? Сука, что за обмудок придумал называть обратимые элементы единицами.
sageАноним 15/06/15 Пнд 23:23:35 #163 №276289 
>>276287
https://en.wikipedia.org/wiki/Unit_(ring_theory)
Аноним 15/06/15 Пнд 23:24:30 #164 №276290 
>>275146
Повторю пост в более подходящем треде:
Аноны, если вкратце - есть математики, которые могут пояснить за переход в полярную систему координат и рассчет площади фигуры в ней?
Аноним 15/06/15 Пнд 23:47:34 #165 №276299 
А в чем суть этих неприводимых элементов кольца? Например, многочлен неприводим, если он не раскладывается на линейные множители. А что означает неприводимость абстрактного элемента кольца? Зачем введено требования, что он должен раскладываться в произведение ab, где либо a, либо b обратим?
Аноним 15/06/15 Пнд 23:55:54 #166 №276306 

>>275923
Ну вот смотри, |A×Q| это мощность множества Q-линейных комбинаций базиса мощностью А. И она оказывается меньше континуума т.к по предположению |A| меньше континуума. А должна равняться ему, т.к мы предполагали что это базис.
Аноним 15/06/15 Пнд 23:57:25 #167 №276307 

>>276022
Конечно. Годнота же. Я даже был на семинаре, где он лично докладывал об этом результате.
Аноним 16/06/15 Втр 00:12:37 #168 №276317 

>>276314
О том что все гомотопические группы (кроме первой) двумерной сферы ненулевые.
Аноним 16/06/15 Втр 00:18:06 #169 №276324 
В чем разница между названиями многообразий в английском variety и manifold? Судя по статье о локальных кольцах это не одно и то же.
Аноним 16/06/15 Втр 00:27:41 #170 №276337 

>>276324
Variety обычно алгебраическое многообразие, manifold - топологическое.
Аноним 16/06/15 Втр 00:28:01 #171 №276339 
>>276324
Ну не знаю насчет твоей статьи, но вот по одному значению:
1. manifold - топологическое пространство снабженное атласом и функциями склейки;
2. variety - совокупность алгебраических структур одного типа, замкнутая относительно гомоморфных образов, взятия подструктур и произведений по произвольным индексированным семействам.
Аноним 16/06/15 Втр 01:53:18 #172 №276378 
Математики, тут могут помочь студенту решить первокурсные задачки с интегралами? Отдельный тред пилить на это не стану, нечего плодить мертворожденные треды.
Аноним 16/06/15 Втр 01:56:41 #173 №276379 

>>276378
Вообще здесь такое не любят. Так как предпочитают угорать по гомологиям, гомотопиям и категориям. Но может кто-то и смилостивится.
Аноним 16/06/15 Втр 02:00:49 #174 №276380 
>>276379
Посоветуешь вывесить обьявление в отдельном треде или бампать этот?
Аноним 16/06/15 Втр 02:02:13 #175 №276381 

>>276380
Лучше скинь сюда.
Аноним 16/06/15 Втр 02:08:38 #176 №276384 

>>276382
Тем не менее это абсолютно новый результат.
Аноним 16/06/15 Втр 02:09:00 #177 №276385 
>>276381
В общем, есть одна методичка с этими самыми заданиями, вот: http://rghost.ru/private/7TbYLZXdY/a1645c1ccf3387165fbdd9343227c659
На весь обьем текста можно внимания не обращать, меня интересует конкретно 19-тый вариант
Аноним 16/06/15 Втр 04:02:40 #178 №276389 
>>276385
И что ты хочешь, чтобы за тебя, уёбка, всё тут порешали? Ты не охуел?
Аноним 16/06/15 Втр 04:18:37 #179 №276390 
>>276379
Вернее сказать, предпочитают угорать по терминам, значений которых даже не знают.
Аноним 16/06/15 Втр 06:53:33 #180 №276392 
14344268131020.jpg
Анон, насоветуй мне годный учебник по теории функций комплексной переменной, чтобы я не завалил экзамен в пятницу.
Желательно чтобы учебник был с небольшим количеством задач с ответами, дабы я мог удостовериться в понимании материала.
Аноним 16/06/15 Втр 08:36:15 #181 №276394 
>>276306
>это мощность множества Q-линейных комбинаций базиса мощностью А
Неочевидно. A×Q - это все возможные варианты одного слагаемого в линейной комбинации.
Аноним 16/06/15 Втр 09:42:10 #182 №276406 
>>276389
БОльшую часть я уже сам опрешал, а вот интегралы несобственные (или как там они звутся) не знаю.
Аноним 16/06/15 Втр 09:49:59 #183 №276407 
>>276406
Умри.
Аноним 16/06/15 Втр 10:55:15 #184 №276425 
>>275146
Как перейти от переполненного базиса к ортогональному?
Как при помощи добавления ребер из квадратной решетки построить сильно регулярный граф?
Аноним 16/06/15 Втр 11:18:42 #185 №276430 
>>276406
>а вот интегралы несобственные (или как там они звутся) не знаю
Надо просто найти неопределенный интеграл, и перейти в формуле Ньютона-Лейбница к пределу, когда переменная, соответствующая бесконечному пределу в интеграле, стремится к бесконечности. Хуй знает как это тебе понятнее объяснить.
мимо добряк
Аноним 16/06/15 Втр 11:42:44 #186 №276433 
>>276299
Анон, объясни. Неприводимый элемент кольца это просто элемент, не разложимый на множители, или надо понимать это определение как-то по-другому?
Аноним 16/06/15 Втр 12:30:49 #187 №276436 
Что значит "продолжение гомоморфизма"? Нигде не нашел точное определение. Вот, например, говорят, что гомоморфизм phi из R в S продолжается до гомоморфизма из R[X, Y, ..., Z] в S. Как это понимать?
Аноним 16/06/15 Втр 12:36:57 #188 №276438 
>>276436
Продолжение отображения знаешь, что такое? Продолжение гомоморфизма - это продолжение отображения, которое является гомоморфизмом.
Аноним 16/06/15 Втр 12:52:44 #189 №276441 
>>276438
То есть, если рассмотреть отображения:
f: A -> B
g: X -> B
то f будет расширением g, если область определения f содержит в себе область определения g, и при этом f(x) = g(x) для всех x \in X?
Аноним 16/06/15 Втр 12:56:39 #190 №276442 
>>276441
Да. Точнее, терминология такая:
f будет продолжением g на область A.
g будет сужением (ограничением) f на область X.
Аноним 16/06/15 Втр 15:27:18 #191 №276448 
>Пусть R — область целостности, т.е. коммутативное кольцо без делителей нуля. Элемент p≠0 называется неприводимым, если он не является единицей, а из равенства p=bc, следует, что либо b, либо c являются единицами.
>если он не является единицей
МРААААЗЬ СУКА хочется убить того мудака, который обратимые элементы начал называть единицами, выворачивая мою интуицию наизнанку. НЕНАВИЖУ ТВАРЕЙ, которые продолжают печатать литературу с таким термином. НЕНАВИЖУ МАКАКУ, которая никак не допилит движок этой говнопараши.
Аноним 16/06/15 Втр 15:29:36 #192 №276450 
Спрошу здесь, пожалуй.
Полтора года назад к 20 годам, заканчивая филфак, понял, что нихера не учил, ничего не знаю и тупой как пробка. Большая часть друзяшек - программисты, в школе паскаль давался более-менее, так что я решил попробовать за полтора года подготовиться к ЕГЭ по математике и информатике, и заодно узнать что-то о погромировании до того, разумеется, у-мамкина-на-шее%. Изначальный уровень подготовки был чуть больше нуля, т.е. что такое дискриминант и квадратное уравнение я знал, но и всё, не больше, вплоть до незнания материала начала 7го класса.
Сейчас, спустя полтора года, тесты ЕГЭ идут на ~65 баллов, их местный украинский аналог ВНО - на примерно такой же балл.
мне нужно будет сдавать вступительные испытания универа
Это вообще нормально?
Сейчас есть альтернатива, но после сдачи экзаменов её уже не будет. А ведь в универе будут ещё матан, etc., и я сейчас не знаю, то ли я тупой и не смог за целых полтора года взять курс школьной математики хотя бы до 80, то ли я не смог всего за полтора года взять курс школьной математики и это ок.
А ведь на подготовку эти полтора года уходило по 3-4 часа в день, деньги. Обидно, сука%%
Аноним 16/06/15 Втр 15:30:58 #193 №276452 
>>276450
Блядь, разметка.
Ну, ладно.
Аноним 16/06/15 Втр 15:31:07 #194 №276453 
>>276450
Ты просто ленивый мудак и не любишь математику. Без интереса в этой области делать нечего.
Аноним 16/06/15 Втр 16:07:25 #195 №276464 
>>276453
Как и в любой. Интерес, это вознаграждение за регулярный труд, а не одаренность.
Аноним 16/06/15 Втр 16:36:19 #196 №276473 
Хочу спросить совета насчёт обучения. Если я учусь на кафедре «Прикладной математики» в МГТУ, но хочу заниматься менее прикладными задачами, смогу ли перевестись куда-нибудь, где будет подобное, или хотя бы после обучения поступить в магистратуру с более подходящим направлением? Что можете посоветовать?
Аноним 16/06/15 Втр 16:39:55 #197 №276474 
>>276473
>учусь на кафедре «Прикладной математики» в МГТУ
Путь закрыт. Даунам-программистишкам даже доказательства теорем не объясняют. Иди свою жаву учи.
Аноним 16/06/15 Втр 16:42:49 #198 №276476 
>>276464
Следует ли из твоего высказывания, что если ты будешь регулярно драить сортиры и сосать хуи, у тебя появится к этому интерес?
Аноним 16/06/15 Втр 16:51:45 #199 №276483 
Поясните за комплексные числа. Если их нет в ряду действительных чисел, то как их вообще представлять? Хуй знает, вроде понятно, что эти числа ПРИДУМАНЫ человеком, но в моей голове не вмещается их полное понимание. Их кто-нибудь вообще понимает? Или "есть они и всё"? Получается, что их сравнивать с действительными числами тоже нельзя?
Аноним 16/06/15 Втр 16:58:29 #200 №276489 
>>276483
Комплексное число - это виртуальное число, которое удобно иногда использовать в качестве промежуточного запоминания неопределённостей, возникающих в некоторых вычислениях и ликвидирующихся при дальнейших вычислениях.
Аноним 16/06/15 Втр 17:01:50 #201 №276493 
>>276489
>Комплексное число - это виртуальное число
А действительное число - реально число?
Аноним 16/06/15 Втр 17:02:11 #202 №276494 
>>276425
>к ортогональному и полному
fix.
Аноним 16/06/15 Втр 17:04:54 #203 №276496 
>>276491
Лал, проиграл с тебя. Ты всегда рандомные фразы кидаешь в ответ на комментарии?
Аноним 16/06/15 Втр 17:16:06 #204 №276499 
>>276474
Объясняют. Опускают только совсем хардкорные (да-да, для вас это мелочи). Дело не в этом. Я хочу примкнуть к более «чистой» математике, и думаю, есть ли сейчас смысл переводиться или лучше закончить, а потом в более подходящую магистратуру поступить. Возможно ли это? Что посоветуете?
Аноним 16/06/15 Втр 17:25:24 #205 №276502 
>>276483
Разрыв очередной жертвы рюзке образования. Это вам так мозги засрали примерами сложения яблок, когда рассказывали про натуральыне и отрицательные числа. Теперь быдло считает, что комплексных чисел нет, потому что ими нельзя посчитать яблоки. Я правильно обучать надо так: математика - сверхъестественная наука, она изучает несуществующее, и все математические объекты существуют внутри нас, а не где-то в мире.
Аноним 16/06/15 Втр 17:34:14 #206 №276504 
Как можно доказать, что если R - факториальное кольцо, то любой неприводимый элемент F из R[X] будет также неприводим над K[X], где K - поле частных R?
sageАноним 16/06/15 Втр 18:15:33 #207 №276509 
>>276483
>то как их вообще представлять
Как точки плоскости. Число x+iy - точка с координатами (x;y).

покормил
Аноним 16/06/15 Втр 18:19:35 #208 №276511 
>>276509
То есть они типа над действительными? Мхм, у меня числа всегда воспринимались как ряд, то есть только слева направо.
Аноним 16/06/15 Втр 18:23:56 #209 №276512 
>>276504
Следует из определений.
sageАноним 16/06/15 Втр 18:24:28 #210 №276513 
>>276511
золотце, иди нахуй
Аноним 16/06/15 Втр 18:34:38 #211 №276514 
>>276512
Погоди, а лемму Гаусса не надо применять? Или что-то еще, связанное с Гауссом.
Аноним 16/06/15 Втр 20:10:43 #212 №276521 
>>276476
Нет, конечно.
Аноним 16/06/15 Втр 20:13:12 #213 №276523 
>>276502
>Теперь быдло считает, что комплексных чисел нет, потому что ими нельзя посчитать яблоки.
Интеренсная мысль, хоть ты и ебанутый.
>>276512
Как и любая теорема.
Аноним 16/06/15 Втр 20:22:30 #214 №276524 
>>276521
Но почему, ты ведь будешь заниматься регулярным трудом, а согласно твоему высказыванию - это должно вызвать интерес?
Аноним 16/06/15 Втр 20:22:46 #215 №276525 
>>276502
Таки натуральные числа используются для соотношения количественных мер. Т.е. 10 овец в поле, 10 ведер воды в стойлах. Человек может посчитать овец, забыть про овец, используя лишь знак числа десяти налить достаточное количество воды. Содержание заменяется знаком, профит и оптимизация.
Но для соотношения каких мер используются комплексные числа?
Аноним 16/06/15 Втр 20:23:40 #216 №276526 
>>276450
в чём заключается вопрос?
Аноним 16/06/15 Втр 20:25:28 #217 №276527 
>>276524
Ты делаешь логическую ошибку. Я говорил, что
>Интерес, это вознаграждение за регулярный труд, а не одаренность.
но здесь не было слов за любой регулярный труд. Реголярного труда недостаточно, но он необходим.
советую тебе быстро схватывать такие тонкости. ИРЛ много пиздаболов и фанатиков используют их в своих целях
Аноним 16/06/15 Втр 20:32:06 #218 №276530 
>>276524
Может ты лучше поймешь мою мысль, что заинтересованность и радость это вознаграждение за труд, если я приведу аналогию. аналогия не аргумент, кстати
Занятия спортом -- это своебразная радость и удовольствие. Тонус, здоровье -- вот это всё. Однако, больному и слабому человеку при занятиях спортом будет только больно и неприятно. Ему нужно долго и медленно повышать нагрузку, вкатываться и подготавливать тело месяцами.
Так и с умственным трудом. У нас есть нервозный, потерявшийся парень, который не может разобраться в себе. Он садится за математику, но получает только разочарования. Почему? Потому что прежде чем работать, нужно иметь ясное сознание.
Аноним 16/06/15 Втр 20:35:58 #219 №276531 
>>276448
Я вроде бы только в коммутативной алгебре помню такое.
Аноним 16/06/15 Втр 20:48:29 #220 №276537 
>>276530
>Драить сортиры и сосать хуи -- это своебразная радость и удовольствие. Тонус, здоровье -- вот это всё. Однако, больному и слабому человеку драить сортиры и сосать хуи будет только больно и неприятно. Ему нужно долго и медленно повышать нагрузку, вкатываться и подготавливать тело месяцами.
Аноним 16/06/15 Втр 20:56:35 #221 №276540 
>>276531
Это у числовиков вроде как такой термин
Вроде бы вполне обоснованный, учитывая что обратимые в кольце целых это те у которых норма 1(?)
>>276504
А это вообще правда?
А есть R - не нормальное(то есть не целозамкнутов поле частных)
А так да лемму гаусса смотри, очень просто доказывается
Аноним 16/06/15 Втр 21:03:43 #222 №276541 
>>275146
Доставьте годных обзорных статей от титанов математики. Прочитал "Архитектуру" Бурбаки, обмалафьился.
Аноним 16/06/15 Втр 21:10:54 #223 №276543 
>>276541
Там же тривиальные вещи, зачем тебе эти обзорные статьи, реальных математических знаний у тебя после них не добавится.
Аноним 16/06/15 Втр 21:12:16 #224 №276544 
>>276543
Доставь, всё равно. Я сам решу, что для меня "реальные математические знания".
Аноним 16/06/15 Втр 21:18:52 #225 №276546 
>>276544
Ну почитай монографию Френкеля - Основания теории множеств. Там всё в таком роде - одна большая обзорная статья.
Аноним 16/06/15 Втр 21:51:42 #226 №276550 
>>276526
Вопрос: я совсем тормоз, если освоил программу всего на ~60-70 баллов за полтора года, или?..
Просто, времени на подготовку уходило и уходит очень много. фактически, получается брутфорс временем.

>>276453
Я на погромиста думаю поступать.
Хотя математика (по крайней мере та, которая есть в школе) мне очень нравится, но я сильно сомневаюсь, что из меня выйдет сносный математик.

>>276530
>Он садится за математику, но получает только разочарования. Почему? Потому что прежде чем работать, нужно иметь ясное сознание.
Ну, кстати, школьная математика его дала, в какой-то мере. Actually, я теперь думаю, что, если бы прошел этот же курс перед гуманитарным образованием, весь филфаковский курс воспринимался бы по-другому.
Аноним 16/06/15 Втр 22:14:24 #227 №276554 
>>276550
>я сильно сомневаюсь, что из меня выйдет сносный математик
Да какая в жопу разница? Математики никому не нужны, это просто занятие для себя, такая игра ума для аутистов.
Аноним 16/06/15 Втр 22:16:28 #228 №276555 

>>276554
Двачую. Просто интересное развлечение.
Аноним 16/06/15 Втр 22:31:22 #229 №276557 
>>276554
Математики не вещи, чтобы быть нужными кому-то.
Аноним 16/06/15 Втр 22:48:37 #230 №276561 
>>276473
Бамп вопросу
Аноним 16/06/15 Втр 23:16:57 #231 №276562 
>>276561
Можешь перевестись. Вопросы в деканате. Можешь поступить после бакалавриата. Иди на матфак ВШЭ. Или, если не получится/хочется хоть на тот же твой мехмат, но с другой специальностью.
Аноним 16/06/15 Втр 23:17:21 #232 №276563 
>>276513
Да ладно тебе, перед тобой вполне может быть великий математический ум. Эйлер тоже не представлял их как пары чисел, комплексную плоскость позже придумали.
Аноним 16/06/15 Втр 23:32:39 #233 №276566 
>>276473
Хочешь заниматься математикой - переводись на матфак. Не хочешь - не переводись лучше, можешь в нму себе ходить и радоваться. Вот у тебя есть целое лето, чтоб подготовиться. Условия перевода там очень хорошие, посмотри.
Аноним 16/06/15 Втр 23:33:20 #234 №276567 
>>276563
Эйлер изобрёл формальные ряды, и это даёт повод считать, что комплексные числа он представлял именно как пары вещественных.
Аноним 16/06/15 Втр 23:38:54 #235 №276571 
>Геометрия на верхней полуплоскости (Лобачевского).
Что это значит? Что под этим понимается?
Аноним 16/06/15 Втр 23:40:06 #236 №276573 
>>276571
Модель геометрии лобачевского есть такая прост.
Аноним 17/06/15 Срд 00:24:07 #237 №276575 
>>276562
А магистратура ВШЭ норм?
Аноним 17/06/15 Срд 00:29:19 #238 №276576 
>>276575
Норм.
Аноним 17/06/15 Срд 00:42:46 #239 №276579 
>>276575
хуйня
Аноним 17/06/15 Срд 00:54:17 #240 №276581 
>>276575
Говно полное.
Аноним 17/06/15 Срд 00:54:33 #241 №276582 
>>276575
Да не, с пивком покатит.
Аноним 17/06/15 Срд 00:55:32 #242 №276583 
>>276575
Тот, кто не закончил магистратуру ВШЭ, никогда не станет элитой.
Аноним 17/06/15 Срд 00:56:01 #243 №276584 
>>276575
Да гавно блять ебаное, у меня там брат вскрылся
Аноним 17/06/15 Срд 01:27:36 #244 №276585 
Анон, что скажешь об упражнениях в этой книге? Хватает ли теории в самой книге для их решения, или нужно хорошо помнить коммутативную алгебру?

www.math.lsa.umich.edu/~wfulton/CurveBook.pdf
Аноним 17/06/15 Срд 02:00:56 #245 №276591 
Математические законы выводятся из наблюдения реальности в такой же степени как физические. Многие физики внесли значительный вклад в математику. Например, геометрия Евклида является прямым следствием малой искривленности пространства. Вот если бы астрономы древности могли наблюдать искривления вызванные черными дырами, возможно, неевклидова геометрия появилась гораздо раньше. Или вот - логика - основа математики, является прямым следствием детерминизма законов классической механики. Вроде бы уже даже придумали квантовую логику, но переводить все на нее никто не торопится. Почему? Потому что основной массив явлений с которыми сталкиваются современные математики управляется классическими законами. Но может быть ситуация изменится если произойдет рывок в квантовых технологиях. Дискас?
Аноним 17/06/15 Срд 08:15:08 #246 №276606 
>>276591
>Или вот - логика - основа математики, является прямым следствием детерминизма законов классической механики.
Лол? Ничего, что логика появилась в IV в. до н.э., а классическая механика в XVII в.?
Аноним 17/06/15 Срд 11:09:45 #247 №276628 
Анон, извини что отвлекаю тебя от гомологий, категорий и прочих илитных споров. Мне бы наверное стоило обратиться в тред тупых вопросов, но тут анон более прошаренный и ответит мне быстрее. Вот есть велосипедный кодовый замок. Три барабана, на каждом барабане цифры от 0 до 9. Объясни мне пожалуйста ананас, как посчитать возможное количество комбинаций для такого замка? Есть ли формула для расчета подобных задач в общем виде?
Аноним 17/06/15 Срд 11:28:00 #248 №276630 
>>276628
Перетолстил. сколько возможных целых чисел от 0 до 999?
Аноним 17/06/15 Срд 11:41:38 #249 №276632 
>>276630
Нее, я не толстил. Я просто очень тупой. Я правда не додумался, что тут все просто - 999 вариантов комбинаций. Я просто помню, что там как то через факториалы подобные задачи считаются. Вроде.
Аноним 17/06/15 Срд 12:26:35 #250 №276647 
>>276639
А как насчет факториалов? Вот нагуглил
>Если в некотором множестве а1,а2....аN переставлять местами элементы, оставляя неизменным их количество, то каждая полученная таким образом комбинация называется перестановкой.
Общее число перестановок из m элементов обозначается Pm и вычисляется по формуле:
Pm=N!
Для кодового велосипедного замка с тремя барабанами какое мы имеем множество? Сколько элементов в множестве будет? 30? 30! не равно 1000.
Аноним 17/06/15 Срд 12:33:10 #251 №276648 
>>276647
Ты не меняешь местами цифры между тремя позициями, ты в каждую позицию выставляешт любую цифру вне зависимости от цифр в других позициях. Для таких условий формула всех возможных комбинаций выглядит так:
КоличнствоЦифр[super]КоличествоПозиций[/super] = 10³ = 1000.
Аноним 17/06/15 Срд 12:34:22 #252 №276649 
>>276648
Разметку не угадал, но, надеюсь, понятно, что я хотел изобразить возведение в степень.
Аноним 17/06/15 Срд 12:46:31 #253 №276656 
>>276648
Да спасибо, теперь все понятно.
Оказывается я еще тупее, чем думал
Аноним 17/06/15 Срд 13:08:10 #254 №276659 
>>276606
Законы то были, просто они еще не были формализованы. Вот если бы физическая реальность была так хитрожопо устроена, что в каждом эксперименте было бы ровно два возможных равновероятных исхода, надо полагать что и логика тогда была бы тогда устроена совершенно иначе.
Аноним 17/06/15 Срд 13:08:22 #255 №276660 
>>276649
https://2ch.hk/faq.html
Аноним 17/06/15 Срд 20:23:49 #256 №276736 
>>276566
Да, хочу заниматься математикой.
Матфак ВШЭ? Какой там вообще контингент людей? И почему именно он, а не какой-нибудь мехмат или пистех? Или там только прикладными вещами занимаются?
>>276562
В принципе, те же вопросы. Неужели матфак ВШЭ ближе всех других факов в этой стране к «чистой» математике?
Аноним 17/06/15 Срд 20:38:01 #257 №276738 
>>276736
В Москве да, по крайней мере если ты хочешь заниматься чем-то алгебраическим.
ИМХО
Аноним 17/06/15 Срд 20:38:36 #258 №276739 
>>276736
Контингент хороший
Аноним 17/06/15 Срд 20:54:35 #259 №276741 
>>276483
Геометрически, как плоскость со стрелочками.
Алгебраически, как выражения с правилами манипуляции.
Аноним 17/06/15 Срд 21:17:10 #260 №276744 
>>276483
Ну оч естественно ващет
1 вариант
Каждое число точка на плоскости, складываются, как векторы, умножаются сложением углов + длина = произведение длин
2 вариант - алгебраический и на самом деле оч естественный
У тебя есть действительные числа, присоединяешь к ним корень из минус единицы стандартным образов: C=R[x]/(x^2+1)
То есть комплексные числа - остатки от деления многочленов на x^2+1, с соответствующими правилами арифметики
Аноним 17/06/15 Срд 21:32:23 #261 №276747 
>>276738
А меня там не выебут сразу после перевода? Если он настолько силен, как его описывают.
Аноним 17/06/15 Срд 21:40:52 #262 №276749 
>>276747
Не там не очень сложно, если тебе это интересно как-то
Аноним 17/06/15 Срд 22:34:00 #263 №276759 
14345696404570.png
Матаны, поясните неразумному абитуриенту за пикрелейтед. Во втором случае неравенство будет всегда верно? Или никогда?
Аноним 18/06/15 Чтв 00:12:22 #264 №276770 
>>276759
Ответ - пересечение решения с ОДЗ. ОДЗ пустое => решений нет
[963610] Аноним 18/06/15 Чтв 00:37:46 #265 №276773 
>>276659
Не понял, законы чего не были формализованы. Ты путаешь причину со следствием: как устроена реальность - как раз вопрос логики. Например, можно поставить вопрос так, что эксперимент либо подтвердит гипотезу, либо нет - ровно два равновероятных исхода.
Аноним 18/06/15 Чтв 01:07:50 #266 №276774 
А накидайте хороших книг по теории вероятностей и математической статистике, что-нибудь попроще для начала.
Аноним 18/06/15 Чтв 01:12:43 #267 №276775 
>>276774
Гугли учебник Черновой.
Аноним 18/06/15 Чтв 01:14:02 #268 №276776 
>>276773
Законы классической механики детерминистичны - повторяя один и тот же опыт каждый раз получаешь одно и то же. В отличии от законов квантовой механики, которые существенно вероятностны. Классическая логика детерминистична как раз потому, что является результатом обобщения наблюдения классических явлений. Если бы были математики живущие в реалиях квантовых явлений, то и логика у них была бы квантовой.
Аноним 18/06/15 Чтв 01:14:36 #269 №276777 

>>276774
Ширяев "Вероятность"
Аноним 18/06/15 Чтв 01:27:14 #270 №276778 
>>276776
Формальная логика — это раздел математики, который занимается именно тем, что если А это Б, то А это С при условии, что Б равно С. Это утверждение не может быть неверным. Нет никакой "квантовой логики".
Аноним 18/06/15 Чтв 01:44:47 #271 №276780 
>>276778
>Нет никакой "квантовой логики".
WOOT?
Аноним 18/06/15 Чтв 01:48:29 #272 №276781 
>>276778
И откуда взялась логика? Это по-твоему априорное знание, которое было бы доступно гипотетическому чистому сознанию без доступа к реальности?
Если логические заключения столь однозначны и не нуждаются в каком-лобо подтверждение, то откуда взялись люди, на полном серьезе, спорившие о том допустим ли закон исключенного третьего.
Аноним 18/06/15 Чтв 07:27:12 #273 №276795 
14346016321000.jpg
>>276780
http://iph.ras.ru/uplfile/chps/quantum-logic.pdf
Решай сам, называть ли это "логикой" или же всё-таки пиздежом.
Аноним 18/06/15 Чтв 11:14:52 #274 №276821 
>>276776
Видимо, ты не внял моему комментарию и все еще путаешь причину со следствием. Законы классической механики детерминистичны, потому что подчиняются законам классической логики, а классическая логика детерминистична, потому что обобщать опыт детерминированным путем практичнее. Нет никаких классических или квантовых явлений самих по себе.
Аноним 18/06/15 Чтв 16:19:41 #275 №276870 
Реквестую годный учебник по матехе, чтобы можно было подготовиться к экзамену. Интегралы и диф. уравнения. Какую книженцую посоветуешь анон?
Аноним 18/06/15 Чтв 16:27:28 #276 №276872 
14346340488070.jpg
Матаны, подскажите формулу для среднего, чтобы близкие, часто встречающиеся значения учитывались сильно, а далекие от среднего почти совсем не учитывались.
Аноним 18/06/15 Чтв 16:39:59 #277 №276873 
>>276872
Просто раздели все числа на десять (сто, тысячу), округли до целых и возьми обычное среднее.
Аноним 18/06/15 Чтв 16:53:44 #278 №276875 
>>276872
Медиана.
Аноним 18/06/15 Чтв 17:09:41 #279 №276878 
>>276875
Вот это подойдет, спс.
Аноним 18/06/15 Чтв 18:59:52 #280 №276889 
Завтра думаю подъехать в букинистический магазин и прикупить книгу по теории вероятностей и математической статистике. Посоветуйте, пожалуйста, авторов каких-то наиболее годных советских учебников по данной теме (статистика интересует сильнее, в частности прикладной аспект: физика, научные работы и т.д.).
Аноним 18/06/15 Чтв 19:39:11 #281 №276894 
>>276889
вербицкий теория веры
Аноним 18/06/15 Чтв 19:41:43 #282 №276895 
>>276894
А если серьезно?
Аноним 18/06/15 Чтв 19:54:35 #283 №276900 
>>276895
Купи ван дер Вардена, если будет. Книжка называется "Математическая статистика", раритетная.
Аноним 18/06/15 Чтв 20:01:32 #284 №276901 
>>276900
Я знаю о существовании ван дер варденской книги, там ее нету. У меня, кстати, есть его "Алгебра" в бумажном варианте.
Аноним 18/06/15 Чтв 20:17:25 #285 №276904 
>>276901
Буржуй. Я тебе завидую.
sageАноним 19/06/15 Птн 00:47:31 #286 №276960 
>>276744
>C=R[x]/(x^2+1)
>R[x]
Никто не заметил толстоты.
Аноним 19/06/15 Птн 11:23:04 #287 №277012 
Итак, господа, очередной вопрос летит итт. Сразу скажу - в вузике не учился, матанчик не учил. Поясните за формулу е^pii+1=0 по-моему. Как показательная функция может быть меньше нуля. Желательно с пояснением того, что это вообще.
Аноним 19/06/15 Птн 11:28:35 #288 №277014 
>>277012
>Как показательная функция может быть меньше нуля.
Там степень мнимая, а у нас есть формула Эйлера, которая говорит, что exp(ix)=cos(x)+isin(x). Поставь в формулу pi, получишь твою формулу. Формула Эйлера выводится, насколько я помню, из рядов Тейлора для exp, cos и sin. В общем, смотри статью Формула Эйлера в рувики, там достаточно.
Аноним 19/06/15 Птн 11:35:21 #289 №277015 
>>276960
Это кольцо полиномов. А что?
Аноним 19/06/15 Птн 12:41:25 #290 №277032 
Аноны, как найти погрешность растрового изображения относительно векторного? То есть, если у нас есть треугольник и его растровое приближение (которое зависит от некоторого параметра - размер ячеек), как посчитать погрешность такого приближения?
Аноним 19/06/15 Птн 12:46:47 #291 №277037 
>>277032
Оцифруй вектор с тем же разрешением и высчитай евклидово расстояние, затем сделай 2x апскейл обоих изображений и опять считай, до тех пор, пока abs(prev_err - err) < допуск. Теоретически, в векторе может вылезти что-то и на меньших разрешениях, но вряд ли.
Аноним 19/06/15 Птн 13:45:07 #292 №277047 
14347107071500.png
>>277033
>>277037
Зря я наверно про изображения сказал, у меня просто похожая задача. В общем, задан математический треугольник, т.е. его координаты. Я строю его аппроксимацию следующим образом, разбиваю область где лежит где лежит треугольник каким-то образом, на равные ячейки. Делаю проверку, если центр этой ячейки лежит в треугольнике, значит он принадлежит нашему треугольнику и так для всех ячеек. Вот, как найти теоретическую погрешность, между площадями треугольника и построенной таким образом аппроксимацией.
Аноним 19/06/15 Птн 14:21:05 #293 №277052 
>>275146
Матаны, нужна формула для подсчёта суммы последовательности:
A, A+1, A+2, …, B-2, B-1, B.
При условии, что A < B.
Аноним 19/06/15 Птн 14:50:13 #294 №277059 
>>277047
>как найти теоретическую погрешность, между площадями треугольника и построенной таким образом аппроксимацией.
Сумма всех площадей твоих ячеек минус площадь треугольника же.
Аноним 19/06/15 Птн 15:30:18 #295 №277073 
>>277059
Так это для конкретного треугольника с конкретными числами с конкретным разбиением, а мне бы теоретически вывести эту оценку, без какой-либо конкретики.
Аноним 19/06/15 Птн 15:33:43 #296 №277077 
а кто-нибудь разбирался в доказательстве слабой проблемы Гольдбаха?
http://arxiv.org/pdf/1305.2897v4.pdf
http://arxiv.org/pdf/1205.5252v4.pdf
Это доказательство - оно специфично для простых?
Или оно как теорема Грин-Тао - работает и в куче других случаев, когда исходная последовательность чисел имеет нужную плотность распределения?
То есть вопрос такой - причём тут простые числа? В чём их особенность в доказательстве? Или особенности никакой нет, окромя того, что они не сильно редко встречаются
Аноним 19/06/15 Птн 15:40:06 #297 №277079 
>>277073
Точнее, я имел ввиду, то как ты предложил, это практическое вычисление погрешности. Но как объяснить эту погрешность, как доказать что она правильная.
Аноним 19/06/15 Птн 15:41:16 #298 №277080 
>>277079
А определение у "правильной погрешности" какое?
Аноним 19/06/15 Птн 15:43:48 #299 №277082 
Вообще вот http://mathforum.org/library/drmath/view/55169.html
Аноним 19/06/15 Птн 15:45:07 #300 №277083 
>>277055
Ответ нужен, а не твои наставления.
Аноним 19/06/15 Птн 16:18:18 #301 №277097 
>>277082
>>277080
Спасибо, вроде, то что нужно. Буду теперь думать как это применить к моей задаче и вообще к невыпуклым многоугольникам.
Аноним 20/06/15 Суб 12:11:30 #302 №277250 
Не кидайтесь хуями, давно интересовал такой вопрос. Вот смотрите, берем окружность, рассекаем в любой точке и вытягиваем ее в отрезок. Возможно ли измерить длину этого отрезка с абсолютной точностью? Я понимаю, что число пи имеет бесконечноеколичество знаков после запятой. Является ли это доказательством того, что идеальных окружностей в реальности не существует? А так же не существует в реальности физических величин, которые можно выразить действительным числом?
Аноним 20/06/15 Суб 12:23:06 #303 №277251 
>>277250
Если ты рассечешь окружность, ты получишь не отрезок, а интервал. А в голове у тебя какая-то каша.
Аноним 20/06/15 Суб 12:57:49 #304 №277256 
>>275209

А дать определение натурального числа ты можешь?
Аноним 20/06/15 Суб 13:02:02 #305 №277258 
Матаны, я ищу учебник по матану который плавно перетекал в фунан. Обложка серого или белого цвета. Помню ещё там было много вставок вроде ОТО, но изложение срогое, на современном уровне. Может кто знает автора или хотя бы подскажет где можно скачать полное собрание всех учебников по матану на русском. На рутрекере смотрел - ничего нет.

PS Это тоно не Фихтенгольц и Кудрявцев.
Аноним 20/06/15 Суб 13:39:57 #306 №277267 
>>276736

> ВШЭ? Какой там вообще контингент людей?

Мажоры и дети чиновников. Готовят смену по дойке этой бензоколонки.
Аноним 20/06/15 Суб 13:42:37 #307 №277269 
>>277012

Это трюк в расчете на лошков. Просто определения всех этих констант подобраны таким образом, чтобы уместитсья в красивую формулу, не более.
Аноним 20/06/15 Суб 13:48:30 #308 №277271 
>>277269
Интересно, что же было подобрано, длина окружности, основание натурального логарифма или единица.
Аноним 20/06/15 Суб 14:16:14 #309 №277275 
>>277267
Ватная картоха закудахтала. Матфак ВШЭ и всё остальное ВШЭ - кардинально разные вещи.
Аноним 20/06/15 Суб 14:51:28 #310 №277281 
>>277271

Вообще говоря всё. Но больше всего способ возведения в комплексную степень.

Аноним 20/06/15 Суб 14:53:26 #311 №277282 
>>277275

Мехмат МГУ всегда был и будет лучшим универом по математике. ВШЭ тупо деньгами балуются, видимо девать некуда. Завели матфак, типа для сурьёзностит, подкупили пару мудаков, вроде вербицкого которые как опущенные бегают по всему инету и лижут им задницу.
Аноним 20/06/15 Суб 14:55:36 #312 №277284 
>>277281
Значение числа пи было подобрано или решение уравнения f'(x)=f(x)? Ну ты молодец, расскажи, как.
>Но больше всего способ возведения в комплексную степень.
Он выводится вообще-то.
Аноним 20/06/15 Суб 14:56:40 #313 №277285 
>>277275

> Матфак ВШЭ и всё остальное ВШЭ - кардинально разные вещи.

Аутотренинг недоматематика из ВШХЭ.
Аноним 20/06/15 Суб 14:57:24 #314 №277286 
>>277284

Иди читай учебник по матану за первый курс. Выводится ему бля.
Аноним 20/06/15 Суб 14:57:50 #315 №277287 
>>277269
Нет, лол. Это естественное следствие общей теории дифференцируемых функций комплексного аргумента.
Аноним 20/06/15 Суб 15:01:47 #316 №277288 
>>277287

Нет.
Аноним 20/06/15 Суб 15:03:17 #317 №277289 
>>277288
Хорошо, тогда скажи, может есть другие комплексно-дифференцируемы продолжения функции e^x на комплексную плоскость?
Аноним 20/06/15 Суб 15:04:48 #318 №277292 
>>277289

Это не имеет значения в данном случае.
Аноним 20/06/15 Суб 15:05:40 #319 №277293 
14348019406800.png
>>277052
Аноним 20/06/15 Суб 15:09:08 #320 №277295 
>>277292
Вся твоя аргументация: "Иди читай учебник по матану", "Нет." и "Это не имеет значения". Похоже шизоида с ГСМ, которому нечего сказать.
Аноним 20/06/15 Суб 15:10:42 #321 №277297 
>>277295

Свою аргументацию читай, я не собираюсь тут распинаться и писать статью на твои обрывки фраз не понятно к чему сказанные.
sageАноним 20/06/15 Суб 15:12:26 #322 №277298 
Анон, я люблю абстрактную дрочильню. Посоветуй сборник задач на доказательства по общей алгебре.
Аноним 20/06/15 Суб 15:12:36 #323 №277299 
>>277297
Моя агрументация - https://en.wikipedia.org/wiki/Euler%27s_formula#Proofs , которую я упомянул тут - >>277014
А твоей аргументации никто не видел.
sageАноним 20/06/15 Суб 15:13:40 #324 №277300 
>>277295
Что за наезд на шизоидов, быдло нормалфажное? Отложил тебе личинку на губы.
Аноним 20/06/15 Суб 15:18:59 #325 №277303 
>>277292
Ты утверждал, что определение комплексного возведения в степень специально подогнано, и то, что комплан здесь не поможет, но тогда ты должен бы был утвердительно отвечать на >>277289. Но так как ты этого не делаешь, ты в дополнение к собственной необразованности расписываешься в своей нелогичности.
sageАноним 20/06/15 Суб 15:19:28 #326 №277304 
>>277298
Алсо, стоит ли рассматривать задачник Кострикина? По-моему дрисня про вычисление по шаблону. Мне нравятся интересные задачи, где надо связывать друг с другом разные понятия и теоремы, чтобы все было связано со всем. Есть ли сборник таких задач разной сложности?
Аноним 20/06/15 Суб 15:30:30 #327 №277306 
>>277299

Во-первых, это не мне адресовалось. Во-вторых, это не аргументация, так как сказанное мною в полной мере относится к формуле Эйлера.

>>277303

Ты подонок и тупая мразь. Комплексная степень по-определению имеет "хорошие" свойства, которые (вместе с свойствами остальных объектов из формулы Эйлера, которые в свою очередь тоже "хорошие" по определению) и позволяют так красиво всё увязать.
Аноним 20/06/15 Суб 15:35:14 #328 №277307 
>>277306
> Во-вторых, это не аргументация
>Ты подонок и тупая мразь.
Шиза продолжается.
Ладно, иди нахуй, математики от тебя не дождешься.
Аноним 20/06/15 Суб 15:35:50 #329 №277308 
>>277306
>"хорошие" свойства
Шизофреник, тебе в /zog.
Аноним 20/06/15 Суб 15:40:09 #330 №277309 
>>277307

Это изначально было не математическое утверждение. Я же не говорю что формуля не верна. Меня всегда раздражал пафос с которым эту формулу преподносят, якобы она связывает два фундаментально разных мира геометрии и алгебры. Хотя скромно умалчивают, что сделано это посредством искусственно созданных мнимых чисел, которые ПО ОПРЕДЕЛЕНЮ наделены нужными нам свойствами позволяющими это сделать.

Аноним 20/06/15 Суб 15:42:46 #331 №277310 
>>277309
>Меня всегда раздражал
Прими вещества и >>277308
Аноним 20/06/15 Суб 15:44:31 #332 №277311 
>>277310

Лучше выебу твою мамку-шлюху.
Аноним 20/06/15 Суб 15:54:34 #333 №277316 
14348048745180.png
>>277286
Проблемы?
Аноним 20/06/15 Суб 15:54:40 #334 №277317 
>>277311
>этот мочератор
Ясно.
Аноним 20/06/15 Суб 15:56:23 #335 №277318 
>>277309
Ну так покажи мнимые числа, не обладающие такими свойствами.
Одно дело то, что в СГС диэлектрическая и магнитная постоянная равны 1 и из-за этого многие формулы простые. Это действительно подобрали, пошаманив единицами измерения. А вот тут что подобрали, мне не понятно. То, что период у exp(ix) sin(x) и cos(x) равен 2pi, а можно ввести my_exp(ix), my_sin(x) и my_cos(x) с другим периодом? Ну так тогда my_exp'=my_exp выполняться не будет. Поэтому то, о чем ты говоришь (ЧТО ИМЕННО подогнали) мне не понятно.
Аноним 20/06/15 Суб 15:56:28 #336 №277319 
>>277316
>Спасибо, Ваша жалоба была отправлена.
Аноним 20/06/15 Суб 15:57:39 #337 №277320 
>>277318
>(ЧТО ИМЕННО подогнали)
 
Все подогнали, жиды же.
Аноним 20/06/15 Суб 15:57:39 #338 №277321 
14348050597050.jpg
>>277319
Аноним 20/06/15 Суб 16:02:51 #339 №277324 
>>277321
Пожаловался на аватарофага.
Аноним 20/06/15 Суб 16:04:20 #340 №277325 
14348054608550.jpg
>>277324
Аноним 20/06/15 Суб 22:10:41 #341 №277387 
Основной трюк в случае связи тригонометрии и экспоненты заключается в подстановке i в ряд для экспоненты, и осознании того что получается сумма ряда для косинуса плюс i на сумму ряда для синуса. Это является следствием только лишь того, что i^2 = -1, однако без вычислений не обойтись. Так что, тем не менее, это нельзя назвать всего лишь "удачно выбранным подогнанным определением".
/discuss
Аноним 21/06/15 Вск 03:58:51 #342 №277422 
>>277387
Вроде же поле комплексных чисел единственное адекватное расширение поля действительных, всё остальное говно получается с делителями нуля и.т.п или я ошибаюсь?
Аноним 21/06/15 Вск 11:03:39 #343 №277440 
Где-то видел книгу с задачами и теорией по тригонометрии для школьников. Помогите вспомнить, что это за книга, хочу немного прокачать знания.
Аноним 21/06/15 Вск 11:11:00 #344 №277441 
14348742606710.jpg
Анон, зачем здесь сделали замену переменной на x и перешли от 2^n - 1 к x^n - 1? Почему мы приходим к заключению, что 2^n - 1 раскладывается на множители? Почему это надо доказывать? Разве это было под вопросом? Ведь есть же формула для разложения таких выражений на множители.
Аноним 21/06/15 Вск 11:14:45 #345 №277442 
>>277441
Точнее, мы раскладываем на множители x^b - 1, но мне все равно непонятно, нахуя делали замену переменных? Почему можно факторизовтаь x^b - 1, но нельзя x^n - 1?
Аноним 21/06/15 Вск 11:17:39 #346 №277443 
Ну так-то я хочу поинтересоваться, а почему произведение комплексных чисел, например, z=(a;b) и w=(c;d) считается по формуле zw(ac-bd;ad+bc), а не как-то иначе? Откуда такая арифметика. Потому как только с такой комбинацией действий числа не будут повторятся?
Аноним 21/06/15 Вск 11:28:49 #347 №277445 
>>277443
Чтобы выполнялись аксиомы поля, уебок.
Аноним 21/06/15 Вск 11:29:54 #348 №277446 
>>277443
Мудакам надо объяснять комплексные числа с помощью изоморфного поля матриц, думаю, тогда у мудаков будет меньше вопросов.
Аноним 21/06/15 Вск 11:40:02 #349 №277447 
>>277422
Ну есть еще, например, неархимедовы упорядоченные поля, расширяющие поле вещественных чисел. Есть теорема Фробениуса про конечномерные расширения поля вещественных чисел и там в самом деле оказываются два варианта - комклексные числа и кватернионы.
Аноним 21/06/15 Вск 11:54:58 #350 №277448 
>>277447

Хаха, петухи кукарекающие весь тред про единственность эпично соснули.
Аноним 21/06/15 Вск 11:55:27 #351 №277449 
>>277258

Бамп вопросу.
Аноним 21/06/15 Вск 12:04:19 #352 №277452 
>>277448
Лол, нет. Поле комплексных чисел в любом случае очень выделенное расширение. В силу того, что оно алгебраическое замыкание и той же теоремы Фробениуса.
Аноним 21/06/15 Вск 12:06:05 #353 №277453 
Аноны, завтра экзамен по линалу, из 54 задач для подготовки решил ~15. Кину несколько задачек, кто-нибудь сможет помочь?
Аноним 21/06/15 Вск 12:20:29 #354 №277455 
>>277443
Потому что i^2 = -1
Аноним 21/06/15 Вск 12:29:10 #355 №277457 
>>277443
Потому что дистрибутивность, запиши в виде суммы и раскрой скобки i^2=-1.
Аноним 21/06/15 Вск 15:19:38 #356 №277495 
http://arxiv.org/pdf/1212.6543v1.pdf
Вот это в шапку пихните.
Аноним 21/06/15 Вск 17:04:02 #357 №277533 
>>277495
Зачем? Человек сформулировал фрагмент ZFC (вроде судя по отзывам теория множеств Цермело с выбором, но я сам не вникал) в категорных терминах, что в целом было известно и до него, но он пытался сделать изложение по возможности ясным широкой математической аудитории. Дело хорошее, но вряд ли что-то всерьез меняет.
Аноним 21/06/15 Вск 17:16:52 #358 №277537 
>>277533
>7. Given f : X → Y and y ∈ Y , one can form the inverse image f−1(y).
Это равносильно аксиоме выбора, да.
Аноним 21/06/15 Вск 19:35:22 #359 №277575 
>>277537
Да. По крайней мере в ZF, но скорее всего у него тоже.
Аноним 21/06/15 Вск 19:38:45 #360 №277577 
>>277537
>>277575
Нет, невнимательно посмотрел, что ты написал. Аксиоме выбора эквивалентно
>10 Every surjection has a right inverse
Аноним 21/06/15 Вск 22:36:34 #361 №277595 
>>277533
>но вряд ли что-то всерьез меняет.
Ты на несколько лет отстал.
http://homotopytypetheory.org/book/
Аноним 21/06/15 Вск 22:56:52 #362 №277602 
>>277595
И что?
Аноним 21/06/15 Вск 23:00:41 #363 №277606 
>>277595
Если что это сильно про разное.
Все что сделано в том препринте - это очередная переформулировка аксиом теории множеств в терминах функций. Это не первая попытка. Может она и изящнее предшественников - сложно сказать, здесь нужно детально вникать в его работу и в собственно более ранние попытки, а у меня нет желания этим заниматься. В любом случае, ничего фундаментального здесь нет.
Гомотопическая теория типов - это совсем другое. Некоторые алгебраические топологи восприняли альтернативный (теоретико-множественному) подход к формализации математики на основе теорий типов. Хотя меня здесь раздражают претензии на революционность и склонность игнорировать своих предшественников, которые имеются у по крайней мере Воеводского. Но этот подход не смотря на всю его любопытность и преимущества (например, каждое доказательство существования дает алгоритм поиска), не был достаточно разработан. Так что, такая реклама, проводником которой ты здесь, кстати являешься, играет и свою благотворную роль.
Аноним 22/06/15 Пнд 01:19:15 #364 №277667 
>>277318
>Ну так покажи мнимые числа, не обладающие такими свойствами.
Какая проблема взять в качестве i другое комплексное число как базисный вектор (если рассматривать С как векторное пространство над R размерности 2 с базисными векторами {1, i})? Все равно будут те же комплексные числа, но формулы будут некрасивые.

На самом деле формула Эйлера не сообщает нихуя полезного. Поворот на 180 градусов (e^iп) == центральная симметрия (-1), вот вся суть этого равенства. Че там где выбрано красиво, что ПААПРИДИЛЕНИЮ так? Просто выбрали НЕ хуевые объекты в качестве базисных, и получили НЕ хуевые формулы связи между ними. Неожиданно?
Аноним 22/06/15 Пнд 01:45:38 #365 №277684 
>>277667
>Все равно будут те же комплексные числа
Нет. Ты несёшь хуйню и не понимаешь, что такое комплексные числа. То есть если я вдруг положу 1=(1,0), i = (0,13), то у меня не получится комплексных чисел. Определяющее свойство мнимой единицы, i^2 = -1, не будет выполняться.
Аноним 22/06/15 Пнд 02:11:42 #366 №277690 
>>277684
>>277684
Лол. Это ты видимо не понимаешь, что такое комплексные числа. В их определении нет числа i.
i,1 это просто координаты. С тем же успехом могли бы выбрать другие.
Аноним 22/06/15 Пнд 02:51:22 #367 №277701 
>>277667
>Поворот на 180 градусов
Ох, лол. Ты перепутал местами причину и следствие, в этом твоя проблема. Есть формула Эйлера, которая показывает, что умножение числа на комплексную экспоненту есть поворот вокруг оси на x радиан:
r exp(ix) = r (cos x + i sin x)
Эта формула доказывается через ряды Тейлора. А далее, используя этот факт, и поставив в нее pi, ты открываешь следствие, что exp(sqrt(-1)*pi)+1=0.

А ты перевернул все вверх дном, как будто "Поворот на 180 градусов (e^iп) == центральная симметрия (-1), вот вся суть этого равенства" - это доказательств не требует, и тогда, оказывается, что это формула не сообщает ничего нового. Конечно, если ты знаешь, что exp(sqrt(-1) pi) - это поворот, то доказательства тебе не нужны, потому что формула Эйлера как раз это и доказывает.
Аноним 22/06/15 Пнд 08:45:11 #368 №277733 
>>277690
В определении комплексных чисел есть число i.
Аноним 22/06/15 Пнд 09:15:01 #369 №277739 
>>277733
Кек, неси свое определение!
Аноним 22/06/15 Пнд 09:35:58 #370 №277740 
14349549589860.png
>>277739
Ты читал учебники когда-нибудь?
Аноним 22/06/15 Пнд 09:43:42 #371 №277741 DELETED
>>277740
> учебники
 
Жидовские выдумки не нужны.
Аноним 22/06/15 Пнд 09:45:26 #372 №277742 
>>277741
Комплексные числа изобрели арийцы.
Аноним 22/06/15 Пнд 09:55:01 #373 №277743 DELETED
>>277742
> арийцы.
 
То есть жиды.
Аноним 22/06/15 Пнд 11:00:46 #374 №277754 
НЕт вы не правы коплексние числа матиматика говно всё говно
Аноним 22/06/15 Пнд 13:59:11 #375 №277789 
14349707511230.png
Решаю задачи из Рудина, не мог решить пикрелейтед. Посмотрев в решебник, я понял, что в условии опечатка: автор хотел написать sn+1 = sqrt(2 + sn). Но вообще, последовательность sn+1 = sqrt(2 + sqrt(sn)) тоже сходится, ее предел примерно равен 1.83. Как его найти аналитически?
Аноним 22/06/15 Пнд 14:01:41 #376 №277791 
>>277733
ты тупой? очевидно можно и не вводить i и не говорить какую-то заумь про минимальное поле. просто запилить (R^2, +, *) соответствующим образом.
Аноним 22/06/15 Пнд 14:05:18 #377 №277792 
>>277791
И чему тогда будет равно sqrt(-1)?
Аноним 22/06/15 Пнд 14:08:26 #378 №277793 
>>277792
<0, 1>, очевидно же
Аноним 22/06/15 Пнд 14:12:19 #379 №277795 
>>277791
Я своё определение показал, теперь ты показывай своё. Посмотрим. Пока что всё выглядит так, словно ты несёшь хуйню.
Аноним 22/06/15 Пнд 14:16:01 #380 №277800 
Математик, а как у тебя работает мышление во время решения задач? Удерживаешь ли ты в рабочей памяти все предыдущие шаги, то есть имеется ли у тебя детальное общее представление о ходе решения со связями между отдельными шагами, или в каждый момент времени ты помнишь только один шаг, над которым работаешь?

Нравится ли тебе вообще рефлексировать и наблюдать, как у тебя создаются мысли в голове?
Аноним 22/06/15 Пнд 14:25:06 #381 №277805 
>>277795
хуле неясного в "запилить (R^2, +, ) соответствующим образом." мне что ли + и здесь расписать?
Аноним 22/06/15 Пнд 14:30:18 #382 №277807 
>>277805
>мне что ли + и * здесь расписать?
Именно. В этом, как бы, вся суть.
Аноним 22/06/15 Пнд 14:36:50 #383 №277812 
>>277807
<a, b> + <c, d> = <a+b, c+d>
<a, b> * <c, d> = <ac-bd, bc+ad>

Доволен?
Аноним 22/06/15 Пнд 14:37:34 #384 №277813 
>>277807
Я понимаю, что в этом суть. Просто этот шаг тривиален, а печатать всю эту парашу с упорядоченными парами не хочу.
//мда, бессмысленные споры и обсуждения - суть легвиона. нихуя тут не меняется.
Аноним 22/06/15 Пнд 14:39:47 #385 №277814 
>>277813
Ну вот я тут >>277812 расписал
Не думаю, что его это устроит. Веруны от математики хуже религиозных фанатиков.
Аноним 22/06/15 Пнд 14:43:29 #386 №277816 
>>277814
Выведи из этого то, что sqrt(<-1, 0>)=<0, 1>
Аноним 22/06/15 Пнд 14:45:39 #387 №277817 
>>277816
Хотя да, из второй формулы следует. В общем получается, что exp(<0,1>*pi)+<1,0>=0, что выглядет еще красивее в целом.
Аноним 22/06/15 Пнд 15:02:05 #388 №277825 
>>277812
При таких формулах "взять в качестве i другое комплексное число как базисный вектор" не получится, i обязано равняться <0,1>.
Аноним 22/06/15 Пнд 15:03:38 #389 №277826 
>>277789

Для начала распиши sqrt(2)=2cos(pi/4).
После применения формулы двойного угла получишь тождество
sn=2cos(pi/2^(n+1)), а предел тут легко подсчитать.

Аноним 22/06/15 Пнд 15:13:10 #390 №277830 
>>277800
К меня от тебя ТРИЗ.
Аноним 22/06/15 Пнд 15:14:48 #391 №277831 
>>277830
>ТРИЗ
Никогда не интересовался подобной хуйней. Развитые с помощью всяких быдлотехник умения атрофируются быстрее, чем развивались. Но меня всегда тянуло рефлексировать.
Аноним 22/06/15 Пнд 15:20:10 #392 №277833 
>>277831
Чужие глубокие паттерны мышления для тебя будут бессмысленными, пока ты не задрочишь те знания, которые эти паттерны выстраивают.
Аноним 22/06/15 Пнд 15:28:03 #393 №277837 
>>277833
>пока ты не задрочишь те знания, которые эти паттерны выстраивают
О каких знаниях ты говоришь? Теоремы и определения математики или способы мышления, которые ты сам открываешь при решении задач?
Аноним 22/06/15 Пнд 15:31:42 #394 №277839 
Чем различаются множества ∅, {∅} и {∅,{∅}}?
Аноним 22/06/15 Пнд 15:35:59 #395 №277841 
>>277839
Количеством элементов.
Аноним 22/06/15 Пнд 15:39:32 #396 №277843 
>>277837
>Теоремы и определения математики
this
Аноним 22/06/15 Пнд 15:40:15 #397 №277844 
>>277837
Второе, вытекающее из первого.
Аноним 22/06/15 Пнд 15:40:21 #398 №277845 
>>277841
Поясни, блять.
Первое - пустое множество. (0 элементов)
Второе - множество состоящее из пустого множества. (1 элемент)
Третье - тоже самое что и второе, не?
Аноним 22/06/15 Пнд 15:40:49 #399 №277846 
>>277839
Структурой.
Аноним 22/06/15 Пнд 15:41:30 #400 №277847 
>>277845
Пустое множество само по себе всё равно остаётся элементом.
Аноним 22/06/15 Пнд 15:43:30 #401 №277848 
>>277845
Пустое множество ∅ - это не пустота. Это пустая коробка.
{∅} - это коробка, в которой лежит пустая коробка.
Аноним 22/06/15 Пнд 15:45:28 #402 №277852 
>>277839
{∅,{∅}} - это коробка, в которой лежат две другие коробки. Первая из них пустая, во второй лежит пустая коробка.
Аноним 22/06/15 Пнд 15:50:43 #403 №277862 
>>277852
>МАТЕМАТИКА - МЫ РАЗБИРАЕМСЯ В ПУСТЫХ КОРОБКАХ
Аноним 22/06/15 Пнд 15:51:51 #404 №277865 
>>277862
Этими коробками производится декомпозиция вполне прикладных задач.
Аноним 22/06/15 Пнд 16:03:41 #405 №277879 
>>277862
Именно. А также в узелках, дырках и стрелочках.
Аноним 22/06/15 Пнд 16:08:26 #406 №277881 
14349785069350.jpg
Наверное платина но все же спрошу, как решить такое?
Аноним 22/06/15 Пнд 17:06:20 #407 №277893 
Недавно дважды пришлось столкнуться с такой задачей, может кто предложит решение:
допустим есть рандомная арифметическая прогрессия:
A = 3 5 7 9 11 13 ...
S = 3 8 15 24 35 48 ... (Si - cумма первых i членов арифметической прогрессии А).
X = 3 11 26 50 85 133 ... (Xi - сумма первых i членов последовательности S).
Так вот у меня никак не получается вывести формулу для вычисления Xi.
Подойдет и программное решение, конечно быстрее чем О(n).
Аноним 22/06/15 Пнд 17:21:22 #408 №277898 

>>277789
Это будет sqrt(2+A), где A - единственный положительный корень уравнения x^4=x+2. Он приближенно равен 1.3532, в итоге sqrt(2+1.3532) действительно приближенно равно 1.83
Аноним 22/06/15 Пнд 17:23:09 #409 №277899 
>>277893
Запомни этот сайт
https://oeis.org/A051925
Аноним 22/06/15 Пнд 17:34:32 #410 №277902 
>>277881
на калькуляторе
1.757932757....
Аноним 22/06/15 Пнд 17:39:01 #411 №277904 
>>277899
все верно анончик. О сайте этом я знаю и последовательность видел, вопрос в том как вывести формулу для любой прогрессии. a1 и d могут быть любыми числами. Эта последовательность и формула для a1 = 3 и d = 2.
Аноним 22/06/15 Пнд 17:40:37 #412 №277906 
>>277902
А где калькулятор найти который корень под корнем под корнем считает? Спасибо.
Аноним 22/06/15 Пнд 17:41:56 #413 №277907 
>>277906
Посчитай от последнего корня в обратном порядке
Аноним 22/06/15 Пнд 18:37:13 #414 №277918 
>>277904
все, решил, кому интересно:
Xi = t(i + 1) a1 + T(i) d , где t(i) - i-e треугольное число, а T(i) - i-e тетраэдрическое число.
Аноним 22/06/15 Пнд 18:41:03 #415 №277921 
>>277800
Я не математик, только учусь.
Стараюсь разбить задачу на части. У каждой части определяю цель. Повторяю рекурсивно, пока подзадача не станет элементарной (т.е. интуитивно очевидной).
Пока получается не очень, но это мой идеал.
Аноним 22/06/15 Пнд 18:44:49 #416 №277923 
>>277918
Интересно. Что за задача-то была?
Аноним 22/06/15 Пнд 19:03:08 #417 №277934 
>>277918
Спасибо за задачку. Несложная, но хоть немножко мозги напряг, а то нихуя не делаю
Аноним 22/06/15 Пнд 20:02:25 #418 №277947 
>>277906
В адресной строке браузера считай.
javascript: var a=0; for (var i=50; i>0; i--) a=Math.sqrt(i+a); document.write(a);
Аноним 22/06/15 Пнд 20:07:13 #419 №277950 
А как иногородние аспиранты Стекловки живут если такие есть, никто не в курсе? Им ведь не дают общежитие?
sageАноним 22/06/15 Пнд 20:17:49 #420 №277959 
>>277947
лучше, наверное alert(a)
Аноним 22/06/15 Пнд 20:23:16 #421 №277964 
>>277959
Да, ты прав.
Аноним 22/06/15 Пнд 20:25:09 #422 №277966 
>>277918
Ты вычислил S. У меня получилось так:
Хn=n(n+1)(n+2)[4a+d(n-1)]/24,
где а - первый член прогрессии, а d - шаг. Первый шаг вычисления А - даёт многочлен второй степени, второй шаг S - даёт уже третий и т.д. можно любую сумму вычислить. Вот интересней вычислить формулу для n-ной итерации, но тут я уже незнаю.
Аноним 22/06/15 Пнд 20:26:06 #423 №277968 

>>277947
Программист что ли?
Аноним 22/06/15 Пнд 20:26:43 #424 №277970 
>>277966
быстрофикс, вернее похожее на S у меня получилась формула
Аноним 22/06/15 Пнд 21:31:01 #425 №277995 
14349978613050.jpg
14349978613091.jpg
Можете какой-нибудь учебник по этому вопроснику насоветовать?
Аноним 22/06/15 Пнд 21:55:21 #426 №278000 
>>277701
>Ох, лол. Ты перепутал местами причину и следствие, в этом твоя проблема.
Это как раз у тебя проблема, потому что
1) Я не говорил ничего о причине и следствии, я говорил, что два факта тождественны, а вот что из чего получается и получается ли - мне похуй
2) Эти два факта не соединены причинно-следственной связью - они оба следуют из формулы
>r exp(ix) = r (cos x + i sin x)
Формула Эйлера - как подстановка п, геометрические соображения - из того, что (cos alpha + i sin alpha) - поворот на угол alpha в комплексной плоскости, что проверяется непосредственно
Аноним 22/06/15 Пнд 22:08:09 #427 №278001 
>>277684
Да, определяющее свойство выполняться не будет. Оно будет иметь другой вид. Но само построенное поле будет изоморфно обычным комплексным числам, и потому все остальные свойства будут такие же - размерность 2 над вещественными, алгебраическая замкнутость, что там еще тебе надо. Обычно структуры определяются инвариантным образом, чтобы не различать структуру и изоморфную ей. Потом уже из списка изоморфных выбирается наиболее удобная, в данном случае - та, у которой определяющее соотношение наиболее простое.
Аноним 22/06/15 Пнд 22:12:43 #428 №278003 
>>278000
Нет, у меня никаких проблем нет, это ты выебываешься на пустом месте.
Аноним 22/06/15 Пнд 22:46:26 #429 №278010 
>>278003
Но у меня тоже нет никаких проблем, это тебе что-то не понравилось, ты придумал себе обоснование своего бугурта и пытаешься с ним теперь до меня доебаться.
Аноним 22/06/15 Пнд 22:49:18 #430 №278011 
14350025581460.jpg
14350025582031.jpg
14350025582612.jpg
14350025583263.jpg
Аноны, порикаминдуйте пожалуйста учебник/задачник по уравнениям математической физики. Пикрелейтед список тем не обращайте внимания на заголовок "Экзаменационные вопросы", я все равно сдавать буду уже в сентябре. Требования к задачнику - задачи Штурма-Лиувилля, метод Фурье, неоднородные краевые задачи, функция Грина, метод интегральных преобразований, преобразования Фурье/Лапласа.
Аноним 22/06/15 Пнд 23:15:03 #431 №278022 
>>278000
>я говорил, что два факта тождественны
>тождественны
Ну охуеть теперь.
Аноним 22/06/15 Пнд 23:41:49 #432 №278028 
>>278011
Самарский?
Аноним 22/06/15 Пнд 23:48:15 #433 №278029 
>>278028
Там разве есть задачи?
Аноним 23/06/15 Втр 00:58:27 #434 №278043 
>>278011
Владимиров уравнения матфизики
но не уверен что этого хватит
Аноним 23/06/15 Втр 00:59:01 #435 №278044 
14350103412700.jpg
>>277950
Бампую
Аноним 23/06/15 Втр 01:37:51 #436 №278049 
>>278043
Задачник неплохой, только нет интегральных преобразований для решения краевых задач. Может еще чего вспомнишь?
Аноним 23/06/15 Втр 08:49:29 #437 №278065 
Помогите мне решить задачу из моей головы.

У нас есть 2 мешка, в одном 20 черных кубиков, пронумерованных с 1 до 20, в другом 20 белых кубиков, так же с цифрами от 1 до 20. Мы совершаем бесконечное число подходов к этим мешкам, вытаскивая 10 рандомных черных, затем 10 рандомных белых кубиков. Сколько мы достанем в среднем пар кубиков с одинаковыми числами за каждый подход?
Аноним 23/06/15 Втр 19:22:12 #438 №278232 
14350765328880.png
>>278065
У меня получилось 5, вроде правдоподобно:
Аноним 23/06/15 Втр 20:04:00 #439 №278245 
>>278232
Спасибо! Можете, пожалуйста, объяснить принцип решения и что Вы тут умножали на что и делили?
Аноним 23/06/15 Втр 20:24:54 #440 №278251 
>>278245
От того, какие номера черных выпали, ответ не зависит - можно считать, что всегда выпадает 1,2...10. Тогда из вытащенных белых половина в среднем попадет в интервал. Отсюда пять.
Аноним 23/06/15 Втр 20:25:53 #441 №278253 
>>278065
Насколько я понимаю, в условии необходимо уточнить, как именно происходит процесс доставания кубиков: достается 10 штук и потом они возвращаются в мешок, или же достается один, сразу возвращается, потом следующий и так далее.
Аноним 23/06/15 Втр 20:27:43 #442 №278254 
>>278253
А почему тогда не так: вероятность достать кубик одного цвета с каким-то числом n на нем за один подход = 0,5.Вероятность достать кубик с числом n из двух мешков одновременно = 0,5^2.
Аноним 23/06/15 Втр 20:33:23 #443 №278258 
>>278254
Парадокс дней рождения
Аноним 23/06/15 Втр 20:38:09 #444 №278261 
>>278251
Теперь все понятно.

>>278253
Достаются 10 черных, складываются в кучу, потом 10 белых, в ту же кучу, потом считаются одинаковые пары.

>>278254
>вероятность достать кубик одного цвета с каким-то числом n на нем за один подход = 0,5
Почему так? Кубиков же не 2, а 20. По-моему, вероятность 1/20.
Аноним 23/06/15 Втр 20:42:04 #445 №278264 
>>278258
А он тут при чем? Можешь развить свою мысль.
>>278261
> Почему так? Кубиков же не 2, а 20. По-моему, вероятность 1/20.
Да, но достается то сразу 10 кубиков, значит это можно приравнять к 10 попыткам.
1/20 10 = 1/2. Для белых и черных одновременно = 1/2^2 = 1/4. А поскольку всего 20 чисел, то 1/4 20 = 5.
Аноним 23/06/15 Втр 20:43:26 #446 №278265 
>>278264
1/20 умножить на 10
1/4 умножить на 20
тут еще и разметка звездочками выставляется, оказывается.
Аноним 23/06/15 Втр 20:49:51 #447 №278266 
>>278264
Да, точно, спасибо!
Аноним 23/06/15 Втр 21:19:25 #448 №278272 
14350835656310.jpg
Матач, на экзамене просили написать общее решение неоднородного диф фура как на пике, где p и q функции от x. Я написал Yоо+Yчн. Потом попросили написать Yоо и тут я затупил. Как выглядит общее решение Yоо?
Аноним 23/06/15 Втр 21:26:00 #449 №278274 
14350839603030.jpg
Хороша книга "Основные понятия алгебры" Шафаревича. А есть что-то похожее и по геометрии с топологией? Т.е. тоже обзорное, кратко обо всём?
Аноним 23/06/15 Втр 21:30:00 #450 №278275 
>>278274
Как извлечь пользу из этой книги Шафаревича? Без базовых знаний алгебры там все равно ничего не поймешь и даже не запомнишь.
Аноним 23/06/15 Втр 22:00:22 #451 №278282 
>>278275
Не знаю, у меня получилось понять кое-что в своё время.
Аноним 23/06/15 Втр 22:28:28 #452 №278286 
>>277995
N.Cutland Computability. An Introduction to Recursive Function Theory
Аноним 23/06/15 Втр 23:08:33 #453 №278291 
14350901137440.jpg
Антоны ! Поясните по-хардкору, как вычислить данный определитель ? Как только не пробовал - ничего не выходит.
Аноним 23/06/15 Втр 23:22:39 #454 №278292 
>>278291
В чем проблема разложить по первому столбцу? Полученные миноры будет легко посчитать.
Аноним 23/06/15 Втр 23:39:30 #455 №278295 
14350919710030.jpg
>>278292
Я вычел из всех строк (кроме первой) последнюю и разложил по первому столбцу. Получилось такой минор. Что с ним делать дальше ? Пробовал вычитать из всех столбцов последний - ни к чему не привело. Что делать ?
Аноним 23/06/15 Втр 23:44:25 #456 №278296 
>>278291
Слева везде нули кроме первой и последней строки? Привести к верхнетреугольному виду в одно действие?
Аноним 23/06/15 Втр 23:45:00 #457 №278298 
Т.е. c1c2c3c4c5(c6-b/c1a)
Аноним 23/06/15 Втр 23:45:27 #458 №278299 
c1 x c2 x c3 x c4 x c5 x (c6 - (b/c1) x a)
Аноним 24/06/15 Срд 00:07:16 #459 №278303 
14350936363220.png
>>278291
Нашел тебе формулу, возможно даже правильную.
sageАноним 24/06/15 Срд 00:08:26 #460 №278304 
>>275146
>математика тред
>нет постов про картошку
сега
Аноним 24/06/15 Срд 00:49:58 #461 №278317 
>>278272
Простого решения для произвольных p ,q нет. Если знаешь одно решение, второе можно выразить через вронскиан. А так - куча частных случаев - Бессель, Матье, гипергеометрическое, Хуессель, полиномы всякие...
Аноним 24/06/15 Срд 02:08:34 #462 №278327 
>>278303
Есть ссылка на источник ?
Аноним 24/06/15 Срд 05:38:00 #463 №278335 
14351134804660.png
Разбираюсь с гугловской статьей googleresearch.blogspot.no/2015/06/inceptionism-going-deeper-into-neural.html , нужен градиент вот этой штуки. Я в принципе написал, но нужно проверить себя, а то оптимизатор не оптимизирует.
Аноним 24/06/15 Срд 11:06:22 #464 №278356 
>>278295
бумага как из жопы
Аноним 24/06/15 Срд 11:08:14 #465 №278357 
>>278272
если p,q произвольные функции x, то общее решение хуйми пойми какое, как повезет тебе на коэффициенты. А так возможно тебя спрашивали, как найти однородное решение, оно находится решением исходного диффура с нулевой правой частью
Аноним 24/06/15 Срд 12:21:04 #466 №278375 
>>278327
Я её вывел. Я же сказал, просто возьми и разложи по первому столбцу, нахер ты что-то вычитать начал?
Аноним 24/06/15 Срд 12:32:55 #467 №278382 
>>278245
Просто считал мат ожидание по определению. С из 20 по 10 - это количество способов вытащить 10 белых кубиков. n кубиков из вытащенных 10 должны иметь номера из вытащенного черного набора, а остальные 10 - n должны иметь невытащенные номера (таких тоже осталось 10 штук).
Аноним 24/06/15 Срд 20:38:12 #468 №278536 
Скиньте гайд по намыливанию трактора от матфековца.
Аноним 24/06/15 Срд 22:09:59 #469 №278551 
>>278357
>>278317
По моему, есть какое-то общее решение. Я уже не помню, но пишется система из двух уравнений и решается. Наверху p и q, а во втором их производные.
Если не изменяет память.
Аноним 24/06/15 Срд 22:11:18 #470 №278552 
>>278536
Этот?
https://gist.github.com/nullst/82b31fb04ff87d037939

Планирую тоже съебывать. Только я учусь в мухосранском мехмате, но мне похуй.
Аноним 24/06/15 Срд 22:15:29 #471 №278554 
14351733296500.png
>>278551
Вот кстати оно. Достаточно было вбить в поиск "Общее решение ЛНДУ". Только под вопросом, всегда ли это даст решение.
Аноним 24/06/15 Срд 23:57:45 #472 №278575 
14351794652820.jpg
>>278552
У тебя публикации-то есть, начальник?
Аноним 25/06/15 Чтв 09:59:44 #473 №278627 
>>277258

Бамп.
Аноним 25/06/15 Чтв 14:49:48 #474 №278663 
господа, есть у кого ссылка на доказательство теоремы ферма-эйлера?
Аноним 25/06/15 Чтв 15:55:01 #475 №278673 
>>278575
Неа, я только поступаю. Это мои влажные мечты.
Аноним 25/06/15 Чтв 17:50:11 #476 №278692 
Итак, господа, спешите рекомендовать мне учебник по теории чисел содержащий исчерпывающий начальный материал. Включающий, например, всю ненужную хуйню, как понятия делимости чисел, умножения целых чисел, теоремы арифметики и т.д. И чтобы не говно мамонта.
С меня 50 тысяч рублей.
Аноним 25/06/15 Чтв 18:05:11 #477 №278693 
>>278692
Спивак
Аноним 25/06/15 Чтв 18:23:27 #478 №278694 
14352458078470.jpg
>>278693
Это арифметика\арифметика2 которые?
Кажется годнота, хоть и написано, что для школьников.
Аноним 25/06/15 Чтв 18:50:15 #479 №278697 
>>278694
Да
Аноним 25/06/15 Чтв 19:32:34 #480 №278702 
>>278673
>Неа, я только поступаю
И как ты поступишь без публикаций?
Аноним 25/06/15 Чтв 20:20:37 #481 №278708 
>>278702
Я видно тебя совсем запутал, очень плохо излагаю свои мысли.
Ситуация такая, что не смог поступить в вышку/мгу, но тем не менее вижу себя в будущем математиком. Думаю, к 3-4 курсу какие-нибудь да статьи у меня будут.
Аноним 25/06/15 Чтв 20:21:42 #482 №278709 
>>278702
Я еще даже на бакалавра короч не поступил, но уже расписал себе план на пять лет вперед, лолд
Аноним 25/06/15 Чтв 20:24:21 #483 №278710 
>>278702
А как вообще написать публикацию студенту? Затронуть интересную тему, поработать над ней, отослать в научный журнал, примерно так?
Аноним 25/06/15 Чтв 20:27:01 #484 №278711 
>>278710
Подойти к преподу по понравившейся теме и спросить. Я один раз сделал ЕОТ домашку в маткаде, а ее препод вписал в соавторы, так у нее публикация появилась. У меня у самого публикация появилась примерно так же - в списке из 10 соавторов, авторов прибора. Но это говно, конечно, а так - не сиди сычом, общайся с людьми.
Аноним 25/06/15 Чтв 20:34:18 #485 №278712 
>>278711
>Подойти к преподу по понравившейся теме и спросить.
Т.е. подойти к преподу понравившейся дисциплине и спросить тему?
>Я один раз сделал ЕОТ домашку в маткаде, а ее препод вписал в соавторы, так у нее публикация появилась.
Кек. Что за домашка такая была?
> У меня у самого публикация появилась примерно так же - в списке из 10 соавторов, авторов прибора.
Расскажи, что за йоба такая. Любопытно, как вообще там всё происходит.
>...а так - не сиди сычом, общайся с людьми.
Таки да, понимаю. Я не тщеславный хикка, возомнивший себя гением.
Аноним 25/06/15 Чтв 21:01:17 #486 №278714 
А при съёбе смотрят на стоящую публикацию, или просто для галки чтобы какая-нибудь была, типа что человек активен? А то у меня тезисы в студенческом сборнике есть, лел.
Аноним 25/06/15 Чтв 21:01:51 #487 №278715 
>>278712
>Т.е. подойти к преподу понравившейся дисциплине и спросить тему?
Ну можешь спросить тему, можешь предложить тему. Говори с людьми, таких студентов любят, а кто не любит, пусть идет нахуй сам.
>Кек. Что за домашка такая была?
Диплом по-моему, записка к нему. Препод дал формулы на листочке, я за сорок минут забил их в маткад. http://cyberleninka.ru/article/n/lazernyy-metod-obnaruzheniya-neftyanyh-zagryazneniy-na-vzvolnovannoy-morskoy-poverhnosti-ispolzuyuschiy-uglovoe-skanirovanie
>Расскажи, что за йоба такая. Любопытно, как вообще там всё происходит.
Да там вообще хуйня была. Я летал на конференцию в ОАО ИСС представлять прибор (http://www.geofizika-cosmos.ru/ru/doc/production/dl_save.php?filename=347k.pdf), в докладе (нихуя не научном - просто описание прибора), было я и 10 главных, которые им занимались, я как алгоритмист, электроник, конструктор, и т. д. и т. п. Я выиграл диплом (скорее всего по политическим причинам - потому что не знаю, что там такого хорошего было в моем докладе), а все дипломные работы публиковались в вестнике СибГАУ. 10 человек, Карл. Так что опубликоваться "паровозом" не проблема, если очень надо.
Аноним 25/06/15 Чтв 21:53:57 #488 №278720 
>>278552
>Задачи несложные и скучные, времени немного. В одной-двух задачках может пригодиться умение отличать секанс от косеканса. Если вы вообще способны запоминать такие вещи хотя бы на один день, то это даст вам пару баллов.
кек может я что то не понимаю но разве запоминание множества формулировок не является обязательным для математика?
(не только оно конечно)

 #489 №2772941
Точнее, мы раскладываем на множители x^b - 1, но мне все равно непонятно, нахуя делали замену переменных? Почему можно факторизовтаь x^b - 1, но нельзя x^n - 1? –
 DELETED
>>277297
Моя агрументация - https://en.wikipedia.org/wiki/Euler%27s_formula#Proofs , которую я упомянул тут - >>277014
А твоей аргументации никто не видел.
Аноним 26/06/15 Птн 01:04:24 #490 №278738 
>>278720
>кек может я что то не понимаю но разве запоминание множества формулировок не является обязательным для математика?
Братан, ну гайд же не о том, как "покорить воробьевы горы", если ты понимаешь о чем я.

>Он перебил меня:
>- Дайте определение окружности.
>Он вел себя довольно агрессивно, и его манера резко отличалась от того, как другие экзаменаторы разговаривали с абитуриентами. Кроме того, другие экзаменаторы никогда не задавали вопросы, прежде чем абитуриент полностью не ответит на вопросы билета. Я ответил:
>- Окружность – это точки плоскости, равноудаленные от данной точки. (Это было стандартное определение.)
>- Неверно!- весело объявил экзаменатор.
>Как это может быть неверным? Он помолчал несколько секунд, а затем сказал:
>- Это все точки плоскости, равноудаленные от данной точки.
>Это прозвучало так, будто мы занимались синтаксическим разбором предложения. Но это было только начало.
>- Хорошо,- сказал он. - Дайте определение треугольника.
>После того, как я дал определение треугольника, он обдумал его, несомненно, пытаясь найти в нем то, к чему он мог бы придраться, а потом продолжил: «А каково определение окружности, вписанной в треугольник?»
>Это привело нас к определению касательной, затем к определению прямой и ко многим другим определениям, и вскоре он уже спрашивал меня о пятом постулате Эвклида о параллельных прямых, который даже не входит в школьную программу! Мы говорили о вещах, которые не имели ни малейшего отношения к вопросам билета и которые выходили далеко за рамки того, что я должен был знать. К каждому слову, которое я произносил, задавались дополнительные вопросы. Каждому понятию я должен был дать определение, и если в этом определении звучало другое понятие, ему я тоже должен был дать определение.
>Спустя час нашей беседы мы перешли ко второму вопросу моего билета. К тому моменту все остальные абитуриенты уже ушли, и аудитория опустела. Очевидно, я оказался единственным абитуриентом, которому необходимо было уделить «особое внимание». Мне кажется, они старались так распределять абитуриентов-евреев, чтобы в аудитории их было не больше одного-двух человек. В ответе на второй вопрос мне нужно было только написать формулу производной отношения двух функций. На этот раз я не должен был приводить никаких определений или доказательств. В вопросе было четко сказано «только формула». Но, естественно, экзаменаторы настаивали на том, чтобы я рассказал им целую главу книги по математическому анализу.
>«Дайте определение производной».
>В стандартном определении, которое я дал, упоминалось понятие предела.
>«Дайте определение предела».
>Потом «Что такое функция?» и все началось сначала.
Аноним 26/06/15 Птн 10:03:45 #491 №278756 
>>278738
Таки я понимаю что вас заботит но получается что в заграничных заведениях математический анти гуманизм? Я сам терпеть немогу эти экзамены решите мол 8 сложных задач за 3 часа, 30 легких за 4 и т.д. Просто формализм встречается повсюду в этой науке, к 5 курсу можно привыкнуть к виду школьных функций.
Или это совсем не надо и тру математик не ответит на вопрос рода что такое автоморфизм мол и так всё понятно оно там само в себя переходит или как-то так.
Аноним 26/06/15 Птн 11:57:43 #492 №278763 
>>277921
Гротендик, ты?
Аноним 26/06/15 Птн 12:35:20 #493 №278765 
>>278763
Почему это ты о нем заговорил. Он продвигал такую концепцию?
Аноним 26/06/15 Птн 12:36:21 #494 №278766 
>>278756
Такое ощущение, будто ты считаешь количесвто ангелов на кончике иглы. Говоришь о какой-то несусветной хуете.
Аноним 26/06/15 Птн 14:32:55 #495 №278775 
>>278756
Речь в пасте о дискриминации евреев. Их валили на вступительных экзаменах в МГУ в конце 70-х, начале 80-х сложными задачами и придиранием к формулировкам. Хотя этот еврейский мальчик знал о математике в школе больше, чем я знаю сейчас.
Аноним 26/06/15 Птн 15:40:14 #496 №278781 
14353224141070.jpg
>>278775
>этот еврейский мальчик знал о математике в школе больше, чем я знаю сейчас
Найс трай мишка))
Аноним 26/06/15 Птн 16:47:03 #497 №278783 
>>278765
Он придумывал огромное количество теорий с большим количеством своеобразных, уникальных слов только для того, чтобы уточнить себе смысл какого-то одного всем известного термина.
Аноним 26/06/15 Птн 18:00:16 #498 №278795 
>>278781
Все знают что мишку не взяли на мехмат. Сугубо из-за дискриминации?
Аноним 26/06/15 Птн 19:32:55 #499 №278800 
>>278783
>Он придумывал огромное количество теорий с большим количеством своеобразных, уникальных слов только для того, чтобы уточнить себе смысл какого-то одного всем известного термина.
Абсолютно, нет, это журналистская мифология.
Аноним 26/06/15 Птн 21:22:39 #500 №278807 
>>278738
Вспомнилось мое позорно участие в "мат боях". Если кто не в курсе - это такие олимпиадки, только там еще нужно свое доказательство защитить перед специально обученным дядькой. Ну так вот, как только я начал рассказывать решение первой задачи до которой я еле допер, он говорит - неправильно. Как неправильно? Где? - подумайте.
Ну еб твою, мать либо говори где ошибка либо иди нахуй, чмо ебаное. И там дается три подхода на задачу. Я их все потратил, и съебал в закат.
Еще была похожая охуительная история с коллоквиума в физтехе, но не знаю, будет ли кому интересно.
Аноним 26/06/15 Птн 21:26:30 #501 №278808 
>>278807
пили про коллоквиум
Аноним 26/06/15 Птн 21:35:42 #502 №278809 
>>278808
В середине первого семестра был коллоквиум по матану. Там нужно было доказывать пройденные теоремки перед семинаристом. Но к нам в гости зашел какой-то важный хуй, уже не помню кто, может быть зав-каф. Начал я рассказывать свою хуйню и дошел где-то до половины. Внезапно у важного хуя нашлось более интересно/срочное занятия чем слушать мою хуйню и он удалился. Тут семинарист мне говорит - давай все сначала, а то я тебя не слушал. И только я сказал пару фраз он выдает это самое злоебучее - неправильно. Что блядь неправильно-то? Важному хую-то было все норм, падлюка ты ебаная. А может просто он не счел ошибку критически важной, в любом случае... Дальше он стал слушать других, и когда был сделан полный круг, он говорит - ну а что у вас с задачей? Так задачу вы решили, а вот теорию не ответили - ну ладно поставлю вам троечку.
Десять лет прошло, а бугурт все еще дает о себе знать.
Аноним 26/06/15 Птн 22:15:26 #503 №278813 
>>278807
Напомни. Дело было давно, но вроде когда я был школьником, на матбоях другая команда должна была вести дискуссию, а жюри давать свои комментарии и оценки после.
Аноним 26/06/15 Птн 22:26:28 #504 №278815 
>>278813
Возможно ты прав, а та штука как-то по-другому называлась, что-то там "защита" или еще чего, я уже точно не помню.
Аноним 27/06/15 Суб 00:36:31 #505 №278824 
14353545916160.png
Ну всё, картошки, вешайтесь пока ромка до вас не добрался. #можемповторить
Аноним 27/06/15 Суб 00:39:19 #506 №278825 
>>278809
а спросить что неправильно не додумался?
Аноним 27/06/15 Суб 00:43:19 #507 №278826 
>>278783
Типичная позиция обывателя. Мол что такое площадь? Ты что дурак задаваться такими вопросами, всем же очевидно и известно, однако как высняется, за этим понятием лежит целая теория, называемая сейчас теория меры. В общем рака тебе и все твой семье выблядков.
Аноним 27/06/15 Суб 01:22:46 #508 №278830 
14353573666560.jpg
/sci/ для ньюфагов
Аноним 27/06/15 Суб 02:58:33 #509 №278839 
>>278826
Ты ему, по-моему, припываешь какую-то позицию. Он же не сказал, что это плохо. Может он сидит и дрочит на схемки каждый день.
Аноним 27/06/15 Суб 03:07:52 #510 №278841 
>>278825
> А что неправильно?
> Думайте
Аноним 27/06/15 Суб 10:28:49 #511 №278871 
>>278824
Жаль, что он ебнутый.
Видимо, недостаточно быть профессионалом в своем деле, чтобы быть человеком разумным.
Аноним 27/06/15 Суб 11:17:34 #512 №278874 
>>278871
Ты чё сука, ты чё
Роман
Аноним 27/06/15 Суб 12:14:44 #513 №278884 
>>278874
Да да, типичный быдлан.
Аноним 27/06/15 Суб 12:31:52 #514 №278886 
>>278884
Ути-пути, мамкина илитка намочила штаники с простого математика :3
Аноним 27/06/15 Суб 13:27:47 #515 №278893 

>>278871
Быть человеком разумным не обязательно, чтобы быть профессионалом в каком-либо деле. А в некоторых случах человеком разумным вообще лучше не быть.
Аноним 27/06/15 Суб 13:51:19 #516 №278896 
>>278886
>А в некоторых случах человеком разумным вообще лучше не быть.
В каких это случаях? Что-то ты запизделся, мой друг.
>>278886
Нет. Меня тошнит от его убогой дугиноподобной философии. Чего только стоит его "матафизическая Новороссия" и прочее пропутинское говно.
Аноним 27/06/15 Суб 14:06:46 #517 №278900 
>>278896
Креаклик порвался. Что, не даёт покоя факт, что все чего-то стоящие люди не скачут, а занимаются реальными делами?
Аноним 27/06/15 Суб 14:47:23 #518 №278904 
>>278896
> "матафизическая Новороссия"
Соус этого можно?

Аноним 27/06/15 Суб 14:58:34 #519 №278905 
>>278904
Почитай его жж. baaltii1.livejournal.com
>Новороссия же - категория нашего мышления, пробужденная, очищенная, она не начинается и не заканчивается, и не сливается уж тем более. Это не полит проект и не территория. Мы уже не будем жить как раньше. Мы изменились, это слить нельзя. Никакой Сурков или Путин не могут это слить при всем желании, потому что это очень внутреннее, очень сокровенное, и очень сильное. Русская Весна чудесным образом показала: кто есть кто. Сбросила маски, сбросила одеяло, сбросила одежды, оголила нервы, психику, людей самих начало выворачивать наизнанку, их истинная природа полезла наружу. Детектор правды и лжи. Так и должно быть. Когда меняются местами реки с озерами, ты трепещешь в определенную сторону, и даже неважно, как ты рассуждаешь умом - все видно на лице. Мой трепет - это слава Новороссии - русской, цыганской, грязной, бедной, лишенной, якобы слитой. Да покоится с миром герой Алексей.
> Всякие ебари вам будут говорить, что вам нужно уезжать на запад. Вглядитесь в их лица, поймите, что лучше сдохнуть на хате на фоне ковра, чтобы кореша завернули в этот самый ковер и похоронили в снегу, чем иметь такое сознание, как у них.
> Вообще хотел бы жить в закрытом СССР, работать в закрытом НИИ, и еще вести драм.кружок в психоневрологическом пансионате, носить одну одежду, есть одну еду, думать о пространстве. Но теперь у меня есть кредитная карточка. Пойду в бутик, куплю себе носки, расплачусь кредитной карточкой. Выгодно, удобно. Хотел бы жить в хрущевке, чтобы полы скрип-скрип, чтобы за окном дымились трубы, чтобы индастриал-ж-ж-ж но ночам, чтобы идти сквозь неподвижность на работу, изучать там пространство и языки, но у меня есть кредитная карточка, пойду возьму кредит и его съем, сожру кредит целиком, пусть врачи потом гадают, что такое с животом, а там кредит.

До кучи всякого говна. То ли это тонкий троллинг, то ли он реально долбоеб.
>>278900
Что ты несешь блять?
Аноним 27/06/15 Суб 15:03:17 #520 №278909 

>>278896
> В каких это случаях?
Например, если ты математик.
Аноним 27/06/15 Суб 15:05:56 #521 №278911 
Посоветуйте учебник по дифгему. Чтобы максимально быстро, не важно если абстрактно.
Аноним 27/06/15 Суб 15:07:01 #522 №278913 
>Мы делаем свою Новороссию, а она делает нас. А белые воротнички из администраций даже не представляют, насколько глубоко наше сознание и осознание. Наша Новороссия - часть Востока, она и здесь, и там. К той физической Новороссией, что сейчас выживает под пулями фашистского режима, у меня сострадание и слезы. К ветеранам спецназа у меня уважение и тоже сожаление, что их довели до такого. Я не такой! Я просто внимательно наблюдал, что делают с русскими в Латвии, начиная с 80-х, заканчивая сегодняшним днем. И понял очень многое. Есть метафизика - она внутри, ее пытаются уничтожить, причем довольно хитро. И тут наступает внутренняя Новороссия, в которой нет никаких белых воротничков. Там тоже много страшного, я увидел ее во сне прошлым летом http://baaltii1.livejournal.com/510016.html Там реально было страшно, но там было глубинное, то, что внутри меня. Не стоит говорить "они". Они - это разные, у всех своя боль.
Аноним 27/06/15 Суб 15:13:17 #523 №278915 
>>278909
Совершенно поверхностное суждение.
Может мы друг друга не поняли? Вот что для тебя разумный человек? Наверное, ты подумал об обывателях. Для меня же это тот, кто руководствуется в своих словах и действиях разумом.
Аноним 27/06/15 Суб 15:30:57 #524 №278917 

>>278915
Ну да, я действительно всё на нормалфагов спроецировал.
Перекат Аноним 27/06/15 Суб 23:48:56 #525 №278959 
>>278958
>>278958
>>278958
Аноним 28/06/15 Вск 16:33:03 #526 №279103 
>>277293
Твой ответ не имеет отношения к вопросу.
comments powered by Disqus

Отзывы и предложения